TEXES ESL Certification Review

Pataasin ang iyong marka sa homework at exams ngayon gamit ang Quizwiz!

BE/ESL Requirements

A district is required to provide: bilingual education program if the district has 20 or more LEP students who speak the same language at the same grade level across the district ESL program if the district has even one LEP student.

Communicative Approach: Comprehensible Input

A hypothesis that learners will acquire language best when they are given the appropriate input. The input should be easy enough that they can understand it, but just beyond their level of competence. Activities Illustrations/Visuals (Visual Narrative) Experiments Cloze Activities Mix and Match Word Sorts

ELLs served by Special Education LPAC/ARD Committee Collaboration

ARD committee needs the LPAC to help identify potential language barriers in the evaluation process LPAC needs the ARD committee to help identify disabilities that may be barriers in the language assessment process

Acquiring a Second Language for School: Cognitive/Academic Development

Acquisition of academic L2 (CALP) closely connected to cognitive development in L1 Skills transfer from L1 to L2 L1 maintenance/L1 literacy in home correlate positively

Support for Students Experiencing Cultural Change

Be attentive Listen Be understanding Show empathy for various cultures Acknowledge difference Create an environment of mutual adaptation When teachers value and encourage the home culture, students' self-esteem is much higher, a stronger identity is developed, and linguistic, social, and emotional adaptation is more successful.

Behaviorism (Skinner)

Brain is a blank slate Imitation of input from environment Habit formation by repetition Errors due to habits Contrastive analysis can predict L2 errors (L1 interferes with L2 acquisition) Audio-lingual methods (in classroom)

How can we promote high cognitive development in ELL's?

By utilizing students' knowledge of their first language to build on their knowledge of the second By capitalizing on what they know rather than be troubled by what they do not know By using the language functions that are important for analyzing, evaluating, justifying and persuading

State-mandated LEP Policies

Commissioner's Rules Concerning Limited English Proficient Students: Chapter 89. Subchapter BB (State Plan for Education LEP Students) Texas Education Code: Chapter 29.063 (LPAC Committees) Texas Administrative Code: Chapter 101.Subchapter AA (Participation of LEP Students in State Assessments)

Promoting Language Acquisition

Comprehensible input (from listening and reading) alone does not account for language acquisition. Output (speaking and writing) must be considered. As with L1, oral language - listening and speaking - generally develops before reading and writing in the second language.

TEKS/ELPS Connection: Writing Language Objectives and Examples of Content and Language Objectives

Content Objective: SWBAT ______________ by ___________________________. (TEKS) (academic task) Language Objective: SWBAT ______________ using _________________________. (ELPS) (specific terms/stems/tools) Examples: Content Objective: SWBAT compare/contrast states of matter by creating a 3 circle chart. (TEKS) (academic task) Language Objective: SWBAT write using matter, mass, volume, and atom appropriately. (ELPS) (specific words/stems/tools) Combined Content and Language Objective Example: SWBAT compare and contrast states of matter in writing, using a graphic organizer and summarizing with new vocabulary.

Components of Sheltered Instruction

Content objectives (grade-level TEKS) Language objectives (ELPS) Background Knowledge Comprehensible input Meaningful activities Variety of strategies and scaffolding Meaningful assessment

Effective Learning Environment

Create a safe environment based on a trusting relationship where support is readily available. Be sure that all children know you believe they can learn. Show interest in children's culture and language; incorporate them into class activities whenever possible. Acknowledge difference; model interest and respect.

Learning Styles

Defined as a biological and developmentally imposed set of personal characteristics that make the same teaching methods effective for some and ineffective for others. Teaching methods should include an auditory, visual, and tactile learning model. These 3 models will meet most students' needs for retention and comprehension. Linguistic (auditory) Mathematical Spatial (visual) Musical Kinesthetic (tactile) Interpersonal Intrapersonal

Formal Assessments

Designed according to rigorous testing theory and principles including field testing. Has established validity - items closely reflect the knowledge or skills to be measured. Has established reliability - gives similar results when retaken.

Alternative Assessment

Developed within the context of the classroom Provides direct measure of student's ability Shows how student learns, the strategies used Reveals higher-order thinking skills: synthesis, inference, etc. Informal - ongoing, performance and content based Authentic - contextualized or "real world" tasks

Why the ELPS?

ELL's benefit from content area instruction that is accommodated to their need for comprehensible input. (Krashen, 1983; Echevarria, Vogt, and Short, 2008) ELL's benefit from academic language instruction integrated into content area instruction. (Crandall, 1987; Snow et. al, 1989) ELL's benefit from programs that hold high expectations for students for academic success. (Samway and McKeon, 2007) Language proficiency standards provide a common framework for integrating lang

TEKS

ELLs are responsible for mastery of the same grade level content area TEKS as native English speaking students. Teachers must teach all of the content area TEKS and develop higher order thinking skills for ALL students regardless of their language proficiency level.

Appropriate Feedback

Emphasize communication and meaning, not correctness. Provide some patterned language to practice specific areas such as verb tense, conjunctions, or transitions. Use modeling - restate student's sentence correctly.

Parent Involvement: Decision-Making

Encourage parents to become deeply involved in the decision-making in their children's schools. Families can help to ensure that the education is culturally and linguistically appropriate by participating as Grade-level volunteers LPAC members Site-based decision making committee members Textbook selection committees Curriculum committees Budget committees and attend School board meetings Make parents aware of opportunities to participate in local, regional, and state professional organizations, such as TESOL, NABE, and Title I Conferences.

Exit from BE/ESL Program

Exit can only take place at the end of the year, and a student may not be exited in PK or K. Parent approval is considered valid until exit criteria are met. LEP students for whom the LPAC has recommended linguistic accommodations on the STAAR Reading or Writing test may not be considered for exit. All criteria for exit must be met based on the current TEA English Proficiency Exit Criteria Chart: Oral language fluency Academic competency Writing proficiency Teacher recommendation Note: Exit cannot be based on TELPAS scores unless indicated in a specific section from the list of approved tests.

Two Types of Assessment

Formal assessments have data which support the conclusions made from the test. Usually referred to as standardized measures. Alternative assessments (also called Informal or Authentic) are not data driven but rather content and performance driven.

Communicative Approach: Language Learning

Formal, conscious process Occurs in formal classroom setting Includes rules for spelling, grammar, punctuation (learned and consciously applied). Some grammatical forms may not develop without explicit instruction. Most effective when focus is on limited number of rules at a time Should not be emphasized in early stages

Language Instruction Step 1: Integrate the Skills

Four domains (listening, speaking, reading, and writing) should be taught in an integrated manner as they are used in authentic communication. Skills reinforce one another

Appropriate ESL Instruction

Grounded in second language acquisition Integrates all language arts skills Listening Speaking Reading Writing Is provided on a continuous and daily basis

Stages of Language Development Two-Word (12-18 months)

Holophrastic strings at first Evolves to clear syntactic and semantic relations (ex: bye bye boat, all gone sticky, here pretty, me go)

Every School District Must:

Identify LEP students Provide the required bilingual education and/or ESL program for identified students Seek certified bilingual and ESL teachers

Examples of Cultural Misunderstanding in the Classroom

If home culture says children only speak when spoken to, they may not volunteer answers in class. If home culture is more interdependent, students may not volunteer answers because it draws attention to the individual. If "wait time" for response is longer in another culture (Asian, for example) than it is in U.S., students may not volunteer answers because they see it as interrupting.

Relationship Between L1 and L2

In L1, oral language development occurs earlier than written language development. In L2, younger learners develop this way, but older L2 learners who are literate in their native language, may develop written L2 more fully before oral L2. English can be developed from written input as well as oral if the input is comprehensible to the L2 learner. L2 learners do not need to be fully proficient in oral English before they start to read or write English. Values and concepts of literacy transfer (print represents a systematic code and carries meaning). Well-developed academic skills and strategies transfer. Concept knowledge in content area transfers.

Difference between ESL and Special Education

It is important to note that ESL is not a remedial or special education program. Special education students have conditions that require appropriate modifications. In most cases, those conditions and the need for the same kind of modifications will continue throughout the student's schooling. ESL accommodates; it does not modify content. Teachers of ESL students need to be continually aware of each student's level of English proficiency and his progress or lack of progress, so that the accommodations that are necessary at the beginning of the school year are not the accommodations needed in the middle or end of the school year. (Adjust accommodations.) It's true that many special education strategies are effective for ESL students, but teachers must constantly monitor and adjust their expectations so ESL students' progress is acknowledged and they are encouraged to achieve more. (Hold high expectations.)

Variables Affecting Motivation/Development of L2

Language learning environment (immersion vs. foreign language) Age (affected by sociocultural, cognitive, personality factors) Cognitive development (the older the L2 student the larger the gap, yet older students may have advantage of being literate and schooled) Cultures of home and school Comprehensible input Social interaction

Nativism / Innatist (Chomsky)

Learn through natural ability Cannot fully utilize language without the presence of other humans Do not require formal teaching The principles of language are possible because they are innate.

L1 Influence on L2 Acquisition

Learners fall back on L1 if forced to use L2 before they are ready Borrowing (incorporation of linguistic material from one language into another) is common throughout most languages Code-switching (alternating between one language and another among bilinguals) is normal

Constructivism (Piaget)

Learning result of social interaction Children construct understanding in context of their activities Early language is egocentric Brain learns when ready Progress from concrete to more abstract, from figurative to operative Exploratory, discovery learning (in classroom)

Stages of Language Development Telegraphic (18-24 months)

Main message words (content) Sentence-like grammar -ing morpheme, in and on prepositions, regular plurals (ex: Andrew want that, No sit there, What that?)

Beginning L2 Readers

May be unfamiliar with English alphabet and spelling patterns Have limited sight word vocabulary Can read predictable text Need more experience with written English If preliterate, need to learn value of reading Need practice to solidify sound/symbol correspondences and aspects of English reading (left to right, top to bottom) It is sometimes not possible to translate a word from one language to another. Translation may result in misinformation. (ex: tools in English has many specific words in Spanish) A word specifying one concept in a language may specify several concepts in another. (ex: en in Spanish means both in and on)

Multicultural Education

Means that the school has the willingness and capacity for success for all students. The teachers have the skills knowledge and strategies to ensure equity of all the students. A multicultural education offers equal opportunity for each child to develop their full potential (pluralism). The teacher facilitates effectively various learning styles and is constantly observing the verbal and nonverbal language of the students.

TELPAS Reading - Grades 2 - 12

Multiple-choice tests administered online Designed around the stages of second language acquisition, while language arts reading tests are designed around grade level reading expectations.

Communicative Approach: Language Acquisition

Natural, subconscious process Occurs when person lives and functions in a language Begins with oral language Natural Order Hypothesis (grammatical structures internalized subconsciously in predictable order, similar in L1 and L2 but not same) Errors part of developmental process

Awareness of Cultural Influence

No one can be aware of all the details of culture that students bring with them. Teachers should always be alert to the possibility of cultural misunderstanding. Realize that lack of participation or any out of the ordinary behavior may have a cultural basis. Don't assume a negative motive for the behavior until you have talked with the student and explored a cultural explanation.

How Schools Can Assist Parents

Offer native language orientation for families about the U.S. school system. Create classes for parents in ESL, literacy, citizenship, and parent education. Family involvement specialists and teachers can facilitate family attendance at orientation nights and parent/teacher conferences. Create family learning projects for parents and students to finish together at home after activity is introduced to students in the classroom.

STAAR L Linguistically Accommodated Assessment

Online, linguistically accommodated STAAR math, science and social studies assessments Available for ELLs who Are not most appropriately assessed with STAAR Spanish, AND Have not yet attained a TELPAS advanced high reading rating in grade 2 or above, AND Have been enrolled in U.S. schools for 3 school years or less starting with first grade (5 years or less if a qualifying unschooled asylee or refugee) Not permitted for an ELL whose parent has denied bilingual or ESL program services Provides more substantial linguistic accommodations than the general STAAR

Examples of Alternative Assessments

Oral interview - teacher asks student questions and makes note of responses Story or text retelling Writing samples Projects/Demonstration - individual or group Constructed response items - written response to openended questions Teacher observation of student attention, response to instructional activity, interaction with other students Portfolios - collection of student work to show progress over time

Helping Parents Promote Literacy

Parents can collaborate in creating books to share with their children. Educators plan a series of meetings where they read illustrated children's books with parents in small groups. Educators model reading skills such as looking a pictures to predict, making inferences, and summarizing.

Acquiring a Second Language for School: Sociocultural Processes

Past Experiences: War, insecure living arrangements, economic situation, amount of past schooling, type of past schooling Current Experiences: Socioeconomic status, home literacy, exposure to standard L2, cultural stereotyping, minority status, patterns of assimilation or acculturation Personal: Self-confidence, self-esteem, lowered anxiety

Scaffolding

Provide substantial amount of assistance and support in early stages of new concepts. Divide tasks into smaller chunks (and be ready to divide again). Plan several rounds of activities for new concepts so students need less teacher guidance and have the opportunity to become more independent.

High-Quality Oral Language Development

Read-alouds and role-playing (using culturally relevant classic and contemporary literature) Legends Riddles Poetry Analogies Idioms/ figurative language

Language Instruction Step 3: Use Task-Based Instruction

Real-life tasks combine languages with non-linguistic function Focus on meaning Requires information gathering, comprehension, interaction, language production

LPAC Responsibilities

Review all pertinent information on all students whose home language survey indicates a language other than English Based on state criteria, determine whether or not the student qualifies as LEP Recommend appropriate language placement Determine appropriate tests and accommodations for state assessment Notify parents of designation and placement Obtain parent permission for placement Keep documentation current (including PEIMS) Annually evaluate each student's progress Facilitate the participation of ELLs in other special programs for which they are eligible Exit students from BE/ESL when they have met state criteria Monitor exited students for two (2) years Notify parents of exit and document notification of exit

SIOP Model

SIOP (Sheltered Instruction Observation Protocol): researched-based model of instruction used to improve academic language proficiency in English language learners. Attention to content objectives, language objectives, background knowledge, interaction, meaningful activities

Required Curriculum for ELLs Chapter 74.4 English Language Proficiency Standards (ELPS)

School districts shall implement this section as an integral part of each subject in the required curriculum. The ELPS are to be published along with the TEKS for each subject in the required curriculum.

Communicating with Parents

Send invitations and announcements in the parent's language. Make contact by phone or in person. Provide school calendar and handbook in primary language whenever possible. Have interpreters available at meetings to facilitate communication, improve comprehension of content, and mediate cultural differences.

Stages of Language Development Preschool and School Age

Simple syntactic structures (3-4 yrs.) Speech sounds correctly pronounced (4-8 yrs.) Improves language deductively Formal teaching introduced Language learning (pragmatics, registers) Most semantic distinctions understood (9-11 yrs.)

Oral Language Development Strategies

Songs Drama Show and Tell Dubbing a video Choral reading Riddles and jokes

Verbal Strategies to help students understand teacher's talk

Speak clearly Paraphrase Repeat key vocabulary in context Summarize main points Limit use of idioms

Factors Contributing to Cultural Bias

Stereotyping: overgeneralization or oversimplification of beliefs about a particular group based on hearsay or limited personal experience Prejudice: Unfair and unreasonable opinion formed without adequate knowledge or thought Ethnocentrism: belief by an individual or group that their beliefs, values, and customs are the only right way. Inability to see value in difference.

TELPAS Texas English Language Proficiency Assessment System

Students are rated based on the annual assessment of their proficiency in listening, speaking, reading, and writing. Designed specifically for all LEP students, including students who are parent denials All LEP students take the TELPAS annually until they meet exit criteria and are reclassified as non-LEP. Reports the progress LEP students make in English acquisition along a continuum of Beginner, Intermediate, Advanced, and Advanced High Every student should make adequate yearly progress by improving a level each year from beginning to advanced high, based upon the Proficiency Level Descriptors (PLDs). Grades K-1: TELPAS includes holistically rated listening, speaking, reading, and writing assessments based on ongoing classroom observations and student interactions. Grades 2-12: TELPAS includes multiple-choice reading tests, holistically rated student writing collections, and holistically rated listening and speaking assessments. The listening and speaking assessments are based on ongoing classroom observations and student interactions.

L2 Reading Conclusions

Students learn to read by reading, so time for reading is crucial. Patterns of graphophonics are discovered, word meanings negotiated and confirmed, syntax acquired and reinforced. Research data supports 4 components of an effective reading program: 1. Large amounts of time for actual text reading 2. Teacher-directed instruction in comprehension strategies 3. Opportunities for peer and collaborative learning 4. Occasions for students to talk about their responses to reading • Reading comprehension is best promoted by extensive exposure to meaningful and varied texts with some explicit phonemic awareness and phonics instruction as well as instruction that develops effective learning strategies for both *decoding (metalinguistic awareness) and *comprehending (metacognitive awareness) text.

Metacognitive Strategies

Students need to develop a mind set that reading is thinking and need to demand that what they are reading makes sense. They need to be able to interact, engage, and construct meaning using different strategies to develop their own comprehension.

TELPAS Writing - Grades 2 - 12

Taken from authentic, performance-based classroom instruction that is grounded in the content area TEKS and ELPS Students are scored based on the Proficiency Level Descriptors (PLDs). TELPAS Writing collections are required to contain at least five writing samples including: At least one narrative about a past event At least two expository texts from math, science, or social studies.

Drawbacks of Formal Assessment

Tends to fragment skills - knowing grammatical structures doesn't mean the student can write May not show whether students really know the material - students may randomly choose correct answers on multiple choice tests "Single-occasion" tests don't necessarily measure a student's competence, only how he performed on that occasion Could be culturally biased; norms are set for students from the mainstream culture

ELLs served by Special Education LPAC / ARD Collaboration - Exit

The ARD committee in conjunction with the language proficiency assessment committee shall determine an appropriate assessment instrument and performance standard requirement for exit...for students for whom those tests would be inappropriate as part of the IEP. The decision to exit a student who receives both special education and special language services from the bilingual education or English as a second language program is determined by the ARD committee in conjunction with the language proficiency assessment committee.

LPAC Language Proficiency Assessment Committee

The Language Proficiency Assessment Committee must be set up and trained by the district. Districts must establish a sufficient number of LPACs to enable them to discharge their duties. Required documentation regarding LPAC: Certificate of training Oath of confidentiality (parent member) Minutes of meeting

Tests for ELLs in Texas

The List of Approved Tests for Assessment of Limited English Proficient Students, Texas Education Code (TEC), §29.056(a)(2), authorizes TEA to compile a list of approved assessments for the purposes of identifying students as limited English proficient for entry into or exit (when appropriate) from bilingual education and/or English as a second language (ESL) programs; annually assessing oral language proficiency in English and Spanish when required; and measuring reading and writing proficiency in English and Spanish for program placement.

ELLs served by Special Education LPAC / ARD Collaboration - Entry

The admission review and dismissal (ARD) committee in conjunction with the language proficiency assessment committee (LPAC) shall determine an appropriate assessment instrument and designated level of performance for indicating limited English proficiency...for students for whom those tests would be inappropriate as part of the individualized education program (IEP). The decision for entry into a bilingual education or English as a second language program shall be determined by the ARD committee in conjunction with the language proficiency assessment committee

The Beginning ESL Writer Needs:

Time to write To write about what he/she knows An authentic purpose for writing To learn spelling, grammar, and mechanics in context Support in reaching beyond expectations Prompt feedback Models for writing.

Drawbacks of Alternative Assessments

Time-consuming to create and evaluate Cannot ensure validity and reliability of results Requires informed judgment to reach sound conclusion about student's learning and student progress

Fluency: Monitoring Student Progress

Track fluency progress by using a graph and/or audio recording. Motivates students to practice fluency Provides immediate feedback Shows progress Work collaboratively with students to record, discuss, and interpret fluency data. Set goals for fluency building.

Stages of Language Development Pre-linguistic

Universal sounds Cries, whimpers, coos Stimulus-controlled, involuntary Not creative, so not language yet

Promoting Oral Language Development in the Classroom

Use vocabulary related to a predictable schedule and repeated classroom routines: roll call, recess, lunch, dismissal. Repeat instructional practices: circle time, literary circles, process writing, group projects.

Currently accepted ESL methods all use these basic strategies:

Visuals - pictures, realia, video Vocabulary development - attention to teaching core vocabulary Active learning - process and apply new content and skills Interaction - use multiple grouping configurations Learning Strategies - help students monitor their own learning

Background Knowledge

What appears to be poor comprehension or memory skills may be lack of schema (prior knowledge) or cultural mismatch. Use what students do know as a starting place. Talk to them about the topic; use a KWL or similar chart. Explicitly point out how past learning is related to the new information.

Community Resources

Work with school counselor and nurse to access community agencies to assist students and their families with physical and mental health needs: immunization, physicals, vision, dental, food bank, "clothes closet", and stress associated with immigration. Work to link students with opportunities to continue their education: For example, helping students contact local community colleges for assistance with the application process, scholarships, and financial aid. Seek out and welcome volunteers to tutor and/or mentor ESL students.

Stages of First Language Development

(Pre-linguistic) 1. Babbling 2. Holophrastic 3. Two-Word 4. Telegraphic 5. School Age

LPAC Membership

(b) In school districts required to provide a bilingual education program, the language proficiency assessment committee shall be composed of the membership described in the Texas Education Code (TEC), §29.063: a professional bilingual educator, a professional transitional language educator, a parent of a limited English proficiency student, and a campus administrator. (c) In school districts and grade levels not required to provide a bilingual education program, the language proficiency assessment committee shall be composed of one or more professional personnel a campus administrator a parent of an English language learner participating in the program designated by the school district

Functions of Languages

-Instrumental - satisfy needs Personal - tell about one's self Interactional - communicate/relate to others Regulatory - control behavior of others Heuristic - question, infer Imaginative - dream, create Informative - inform, educate

Guidelines for Alternative Assessments

1. Decide what students should know and be able to do at the end of the lesson or unit. 2. Set the criteria for excellent performance and for minimum passing performance. Create a scoring rubric. Unless language conventions are the purpose of the lesson, scoring criteria should be based on evidence of content knowledge not language proficiency. 3. Select an alternative assessment or allow student choice of several options. Use options that appeal to different learning styles: an analysis, a practical solution, an artistic interpretation, etc.

2 types of ESL programs

1. ESL/content based: English language learners are given ESL content support services within the content areas with an ESL certified teacher. 2. ESL/pull out: English language learners are given ESL support. They are taught basic vocabulary and language structure, usually outside their homeroom classroom and are then integrated into all English instruction for the rest of the day.

Language Register Categories

1. Frozen (ex. "Happy Birthday", "The Pledge of Allegiance") 2. Formal - standard English 3. Consultative - less direct than formal English 4. Casual - general vocabulary, maybe poor syntax 5. Intimate - language of lovers and twins

Cueing Systems

1. Graphophonic - letters/sounds & visual clues What would you expect to see? 2. Semantics - sense, meaning Does it make sense? 3. Syntax - structure, grammar Can we say it that way?

Problems with Word Recognition (Phonics Instruction First)

1. Idea that written language develops only after oral language is mastered delays reading. 2. Pronouncing a word is not same as understanding its meaning. (ex: homonyms, multiple meaning words) 3. Assumption that if students can identify words, they can put meanings together 4. Danger in assuming that a child who pronounces words easily is comprehending

L2 Writing

1. L2 learners have limitations in vocabulary, syntax, idioms. 2. L2 learners have confidence and skill gained in L1 writing. 3. Opportunities for L2 students to write enhance L2 acquisition. 4. Strategies effective in teaching L1 writing are effective when accommodated for L2 learners. (ex: process writing)

Ideas About Print (all emergent readers - L1 and L2 - must grasp)

1. Print carries meaning, conveying a message. 2. Spoken words can be written down and preserved. 3. Written words can be spoken, that is, read out loud. 4. In English, words are read from left to right, top to bottom. 5. In English and other languages that use alphabets, the speech stream can be divided into sounds, and these sounds are represented by letters or groups of letters (grapho-phonemic units). This is the alphabetic principle. 6. The speech stream has a linear sequence in time that corresponds to written languages' linear sequence on the page. 7. Sound/symbol correspondences are consistent, but in English, there are many exceptions. Ideas about print are learned through immersion in a literacy-rich environment with multiple opportunities to hear stories and to write. Thus, teaching strategies should emphasize immersing students in meaningful, functional uses of reading and writing combined with explicit instruction to assist students to become independent readers and writers.

Semantics + Syntax

1. Provide support to make sense of text. 2. Call attention to way language is used in text. 3. Discuss meaning and interpretation of sentences and phrases in text. 4. Point out words that have been encountered before. 5. Help readers discover grammatical cues that indicate relationships such as cause and effect, antecedence and consequences, comparison and contrast, etc.

Scaffolding

1. Teacher does. Student watches. 2. Teacher does. Student helps. 3. Student does. Teacher helps. 4. Student does. Teacher watches & is ready to step in with support when necessary.

4 types of Bilingual Programs

1. Transitional bilingual/early exit: English language learners receive a portion of their content instruction in their primary language for one to three years and then are integrated into all-English instruction. 2. Transitional bilingual/late exit: English language learners receive content instruction in both L1 and L2 for four to six years. 3. Dual language immersion/two way: Biliteracy program model that integrates English proficient students with English Language Learners. Academic subjects are taught to all students through both English and the other language. 4. Dual language immersion/one way: Biliteracy program model that serves only English Language Learners. Academic subjects are taught in L1 and L2 (English).

Areas of cultural difference

1. View of the self: a. Independent Self - "I" - U.S. Americans of Western European heritage. b. Interdependent Self - "We" - Native Americans, African-Americans, Hispanics, Asians. 2. Sense of time: a. Focus on past b. Focus on present c. Focus on future 3. Uncertainty avoidance: a. Difference is curious b. Difference is ridiculous c. Difference is dangerous

Meyers vs. Nebraska 1923

14th Amendment provides protection for language minorities

Unitary stage

24 months speech is abbreviated child uses two word utterances

Cognitive Strategies and why are they important?

3 Categories: -Rehearsal -Organization -Elaboration Examples: •Identifying similarities & differences •Note taking •Reading Comprehension Strategies •Vocabulary Strategies •Visualization •TPR •Brainstorming techniques Why're they important? Strategies represent the dynamic processes underlying learning Active learners are better learners Strategies can be learned Academic language learning is more effective with learning strategies Learning strategies transfer to new tasks

intermediate fluency

3-5 years -has excellent comprehension -makes few grammatical errors

structural awareness stage

60 months child makes errors by overgeneralizing

Discourse

A continuous stretch of speech or written text, going beyond a sentence to express thought. For example: style in writing or rules of conversation Cultural rules for conversation ex: taking turns, opening conversation (How are you?) Differences between written and spoken discourse Rhetorical patterns - ways information is ordered in text ex: linear vs. curvilinear Cohesion Devices - help connect sentences together in a text Reference (ex: pronouns) Conjunctions (then, therefore, afterwards, etc.) Substitution (ex: I do too.) Lexical cohesion (I opened the box. The lid broke.)

TELPAS Holistic Rating

A holistically rated assessment process which is performance-based (using the Proficiency Level Descriptors—PLDs) is used for the following grades and language domains of TELPAS: Grades K-1: listening, speaking, reading, writing Grades 2-12: listening, speaking, writing

Sheltered Instruction

A means for making grade-level academic content (e.g. science, social studies, math) more accessible for English language learners while at the same time promoting their English language development.

Morphemes

A morpheme can be one syllable (pig) or more than one syllable (elephant). It may be a whole word or a part of a word. There are two basic types of morphemes: free and bound. Free morphemes Bound morphemes Free Morphemes Base words Like Differ truth Compound words Light + house =Lighthouse Bound Morphemes Affixes Prefixes Anti Re Suffixes ed ing

Reading Process

A reader uses linguistic and background knowledge while interacting with print. The good reader has automatic word recognition so he/she can concentrate on comprehension.

Lexical Ambiguity

A situation in which a word has two or more meanings Examples: On my way to the bank to cash my paycheck, I passed by the park and saw the most colorful ducks swimming by the bank of the river. While taking notes about the farm, I dropped my pen into the pen where the pigs were playing in the mud.

Syntactic Ambiguity

A situation where a sentence may be interpreted in more than one way due to ambiguous sentence structure. Examples: I gave a few olives to my friend that I stabbed with a fork. Fix it: I gave a few olives, which I stabbed with a fork, to my friend. Sitting quietly on the porch swing, the wasp startled the old man. Fix it: Sitting quietly on the porch swing, the old man was startled by the wasp.

Dialect

A variety of a language defined by both geographical factors and social factors, such as class, religion and ethnicity.

A second-grade teacher has several beginner and intermediate ELLs in the class. To help the ELLs become proficient in English, the teacher knows to make the content comprehensible. According to the SIOP model of instruction, which THREE of the following strategies are most appropriate to meet the teacher's objective? A. slowing down speech and enunciating clearly B. paraphrasing the Ell's response in correct English C. copying notes in English for the ELLs D. using wait time to allow ELLs to express themselves E. having the ELLs complete a journal entry after each lesson

A, B, D

ELPS framework

A. Introduction Required curriculum Integrate social and academic English in content areas Apply to K-12 B. District Responsibilities Identify student proficiency levels Linguistically accommodated content instruction: Communicated, Sequenced, Scaffolded Content-based language instruction Instruction for B/I ELLs: Focused, Targeted, Systematic C.Student Expectations Learning strategies Listening Speaking Reading Writing D. Language Levels (Proficiency Level Descriptors) Beginning Intermediate Advanced Advanced High

A high school English teacher is about to start a unit on Jack London's "To Build a Fire" in her sheltered class. She starts a whole-class discussion by asking: "When the temperature is very high and you feel very hot, what are some things you are supposed to do to stay safe?" She writes their comments on the board and encourages students to follow up on some of the things they say. Then she tells students to work in groups to respond to this question: "When it's very, very cold, what are some things you should do to stay safe?" She gives students five minutes to prepare their group response. How does this oral language activity promote students' communicative language competence? A. The teacher activates prior knowledge to help students connect their real-world experiences to the context of the classroom lesson. B. Using the students' group responses, the teacher will be able to create a semantic map to introduce the story. C. The teacher knows this story will be challenging for ELL students, so she starts with this activity in order to boost their comprehension. D. The teacher wants to promote students' active listening skills by making them realize how little they know about extreme cold temperatures.

A. The teacher activates prior knowledge to help students connect their real-world experiences to the context of the classroom lesson. Rationale: Moving from hot to cold temperature extremes in this prereading activity is an effective approach in this sheltered class. The teacher is providing abundant scaffolding for meaningful discussion and activation of prior knowledge.

A high school biology teacher is starting a unit that addresses state assessment standards on knowledge of interactions among biological systems in plants. Which of the following instructional activities would most effectively promote his ELL students' achievement in this area? A. The teacher assigns a group project. Each group identifies a specific plant and uses visuals and props to demonstrate interaction of its biological systems. B. The teacher has students fill out a questionnaire about their prior knowledge on plants' biological systems. C. The teacher takes his students on a walk around the campus to point out different types of plants growing on the school grounds. D. The teacher shows students the state assessment standards in biology and explains the concepts and defines all the content-specific terms in each standard.

A. The teacher assigns a group project. Each group identifies a specific plant and uses visuals and props to demonstrate interaction of its biological systems. Rationale: This response calls for a great deal of learner participation in a collaborative project that will take individual interests and abilities into account as well as provide hands-on learning.

Mr. Sato, a middle school science teacher, sees that the ELLs in his class are having difficulty understanding a new unit about volcanoes. Which TWO of the following will be the most effective strategies to help his students understand the content? A. asking the students to write down what they know about the day's topic at the beginning of the class B. giving students an English dictionary so they can write definitions of key vocab before each lesson C. pairing an ELL with a native English speaker so students can read the text together D. having students preread selections at home prior to each lesson or activity E. showing a short video clip about the day's topic at the start of the class

A. asking the students to write down what they know about the day's topic at the beginning of the class E. showing a short video clip about the day's topic at the start of the class

An ESL teacher uses supplementary texts in class that feature aspects of different cultures. The teacher also highlights vocabulary words from each unit that can be translated into the two or three languages the students speak. The class reads the words together and discusses them before beginning each unit. The teacher's actions are primarily examples of which of the following? A. culturally responsive instruction B. cultural relativism C. ethnocentrism D. multilingual education

A. culturally responsive instruction

Which of the following is most important for effective second language acquisition? A. engaging in social activities and meaningful interactions B. memorizing and practicing frequently used words and phrases C. interacting primarily with the teacher and other adults D. learning with various structured and rote activities

A. engaging in social activities and meaningful interactions

Planned ESL instruction should ideally focus on which of these? A. listening, speaking, reading, and writing English at every proficiency level B. primarily in speaking English at each level with secondary focus on reading C. mainly speaking and listening with secondary focus on reading and writing D. mainly reading and writing English and then listening and speaking English

A. listening, speaking, reading, and writing English at every proficiency level

An elementary teacher has several ELLs at the beginning level of English language proficiency in the class. The teacher is beginning a unit on volcanoes. The most effective strategy for the teacher to use to make the content comprehensible for the ELLs is to: A. preteach the unit vocab using visuals B. read the unit to the students as they follow along silently C. place the ELLs in a group to read and discuss the textbook info together D. having the ELLs take a multiple choice pretest about volcanoes

A. preteach the unit vocab using visuals

Which of the following activities will best help develop the oral language proficiency of elementary ESL students? A. providing wordless picture books for the students to narrate B. having students chant a poem after teacher modeling C. preparing a listening center for students to listen to audio books D. encouraging students to record themselves reading a story

A. providing wordless picture books for the students to narrate

An ESL teacher works with a class of ELLs who read with various levels of proficiency. To assist with students independent reading, the teacher assigns individual reading assignments on computer. Which of the following statements describes the most important benefit of utilizing technology in reading instruction? A. students receive immediate feedback on their reading performance B. students have access to a wider range of reading genres C. students have more opportunities to increase their vocab development D. students will develop a greater appreciation of literature

A. students receive immediate feedback on their reading performance

A teacher is explaining to an ESL class that "She knew what not to do" indicates something completely different from "She knew not what to do." Into which language area does this distinction primarily fall? A. syntactic B. academic C. pragmatic D. semantic

A. syntactic Rationale: the word order in each sentence is different, changing the meaning of each

Which of the following best characterizes the education of language minority students in the US before WW2? A. there was no concerted effort to assist ELLs in school B. ESL programs were common in larger urban school systems only C. students who did not speak English could be legally prevented from registering in school D. the majority of ELLs with limited English proficency

A. there was no concerted effort to assist ELLs in school Rationale: were expected to assimilate with no assisted English instruction

ELPS Learning Strategies

A. use prior knowledge and experiences to understand meanings in English; B. monitor oral and written language production and employ self-corrective techniques or other resources; C. use strategic learning techniques such as concept mapping, drawing, memorizing, comparing, contrasting, and reviewing to acquire basic and grade-level vocabulary; D. speak using learning strategies such as requesting assistance, employing non-verbal cues, and using synonyms and circumlocution (conveying ideas by defining or describing when exact English words are not known); E. internalize new basic and academic language by using and reusing it in meaningful ways in speaking and writing activities that build concept and language attainment; F. use accessible language and learn new and essential language in the process; G. demonstrate an increasing ability to distinguish between formal and informal English and an increasing knowledge of when to use each one commensurate with grade-level learning expectations; and H. develop and expand repertoire of learning strategies such as reasoning inductively or deductively, looking for patterns in language, and analyzing sayings and expressions commensurate with grade level expectations.

A ninth-grade speech communication class is evenly divided among ELL students and native speakers. The teacher has assigned an informal speech. As part of the preparation for this assignment, she shows several movie clips with the sound turned off. The rational that best explains how this activity will promote student achievement in this informal speech assignment is that A. watching film segments with the sound off will provide an opportunity to analyze how nonverbal cues, body language, and gestures contribute to communication. B. watching a film prior to a challenging assignment reduces anxiety and helps students perform at a higher level. C. watching film clips with the sound off and then rewatching with the sound on will enable students to recognize how important good elocution is in being understood by an audience. D. watching film clips will show students how important staging is in delivering a good speech performance

A. watching film segments with the sound off will provide an opportunity to analyze how nonverbal cues, body language, and gestures contribute to communication. Rationale: . This "silent-viewing" activity is a common strategy for helping students understand how nonverbal cues contribute to communication. In an ELL context, this is an excellent strategy for helping students recognize the meaning provided by nonverbals in L2.

Stages of Acculturation (4 stages)

Acculturation: A general term for the process of becoming adjusted to another culture. STAGE 1: Euphoria - excitement and happiness with new place and new experiences STAGE 2: Culture Shock - realization of the depth of difference between home culture and the new culture; may cause frustration, anger, depression STAGE 3: Tentative Recovery - finding ways to cope with the new culture; forming friendships and support systems STAGE 4: Assimilation or Adaptation - a choice Assimilation - deciding to become like members of the new culture, to accept their surface and deep culture; giving up the original culture Adaptation -finding ways to adapt to and become part of the new culture while maintaining important values and customs of the original culture

Questioning Continuum

Ask questions before, during and after reading. Simple Questions: Who? What? Where? When? Responses: Responses should • Focus on the information on the page. • Rephrase text that has just been read. • Recall facts, events, and names (labeling). Complex Questions: How? Why? What if? Responses should • Move away from what can be seen on the page (making inferences). • Analyze characters' actions and motivations. • Focus on vocabulary and concepts. • Connect story and students' experiences.

Uses of Assessment

Assessments are used for: Identification Program Placement Annual Assessment Formative Assessment Gifted/Talented Exit

According to the English Language Proficiency Standards (ELPS) required by the Texas Administrative Code, curriculum requirements include that school districts offer one of more courses in required curriculum containing a research writing component for students entering grade... A. 12 B. 9 C. 6 D. 3

B. 9 Rationale: a research writing component must be included in courses offered in the required curriculum for students entering grade 9

To promote her fifth-grade ESL students' academic language proficiency, a science teacher takes her students to the school library once a week and has the students check out books on topics related to the unit they are currently studying. The teacher notices that students talk constantly in the library, showing each other their books, and reading each other's books. She recognizes this as an opportunity to promote her students' communicative language development. Which of the following instructional activities best addresses the teacher's intent? A. Assigning students book reports on the books they pick and posting the reports on the class writing wall B. Having each student do a book talk on his or her book C. Having each student post the title of her or his book on the class notes wall. D. Having a question-and-answer session where the teacher asks each student one question about the book he or she checked out

B. Having each student do a book talk on his or her book Rationale: Book talks are a recommended strategy for promoting ELL students' oral-language proficiency. This scenario additionally demonstrates the use of a book talk in a content-area lesson

Which of the following programs provides federally mandated accountability data on the progress of ELL students in Texas in meeting language proficiency goals? A. The State of Texas Assessments of Academic Readiness (STAAR) B. Texas English Language Proficiency Assessment System (TELPAS) C. Texas Essential Knowledge and Skills (TEKS) D. English Language Proficiency Standards (ELPS)

B. Texas English Language Proficiency Assessment System (TELPAS) Rationale: TELPAS includes criteria for rating students' yearly language proficiency as beginning, intermediate, advanced, or advanced high. (A) is the state's assessment system for language arts, math, social studies, and science. Accommodations for ESL students are included in STAAR, but it does not target ESL proficiency.

Each Monday, a third-grade teacher introduces new content vocabulary to her beginning ESL students. To reinforce their initial understanding of the new words, she posts labeled pictures of the words. She wants to promote their listening and speaking proficiency in the context of content instruction. Which of the following instructional activities most effectively addresses the teacher's goal? A. The teacher puts up the lists of new content words, pronounces each one, and then calls on student volunteers to pronounce them, too. B. Working in groups, students pick one of the new words to explore by looking in their books, using the dictionary, and using other class resources. Each group does a short presentation to introduce the class to the new word. C. The teacher gives students two days to learn the words. They spend a few minutes each day pronouncing the words out loud in unison. On the third day, they have a spelling test on all the new words. D. The teacher shows an animated video in which animal characters introduce the new words on the list. After the video, the teacher gives students a short test to determine which words they seemed to understand best.

B. Working in groups, students pick one of the new words to explore by looking in their books, using the dictionary, and using other class resources. Each group does a short presentation to introduce the class to the new word. Rationale: This response shows attention to the teacher's goal to reinforce content-area learning through oral-language proficiency. Working in groups will promote listening and speaking proficiency. The presentations create a learner-centered context for further promoting listening and speaking skills.

"The students, who had studied hard for their examination, which was given at the end of the school year." Based on the sample, the student writer is struggling to... A. make the subject and verb of a sentence agree B. write a sentence containing a subject and a predicate C. place a subordinate clause after the word it modifies D. use the correct pronoun to begin a subordinate clause

B. Write a sentence containing a subject and predicate. Rationale: multiple subjects, but no predicates present

An ESL teacher observes ELLs struggling to comprehend reading passages. Which of the following strategies will best help the ELLs increase their comprehension of a text? A. Asking the ELLs to write notes as they read a new passage B. building ELLs background knowledge before presenting a new passage C. assigning ELLs reading passage for homework as additional practice D. having the ELLs answer multiple choice questions after reading a new passage

B. building ELLs background knowledge before presenting a new passage

A high school history teacher wants his ESL class to develop a deeper understand of historical events. He develops the following list of essential questions: • Who are the pivotal participants? • How does each participant impact the outcome? • Why is this event important? • Could the participants have taken any other course of action? Following each unit, students discuss the questions in groups and make brief oral reports on their findings. This instructional strategy promotes students' content-area proficiency by A. integrating several levels of Bloom's taxonomy. B. engaging students in critical thinking through activities that foster communicative competence. C. impressing upon students the need to memorize the names of key historical figures. D. encouraging students to create a historical timeline that helps them remember when pivotal events happened.

B. engaging students in critical thinking through activities that foster communicative competence. Rationale: The essential questions and oral presentations promote critical thinking in the context of communicative competence

An elementary teacher has a class of beginning ESL students. During oral reading times, the teacher notices that the students are using many L1 phonemes in pronouncing L2 words. The most effective strategy in helping students use their L1 phonological knowledge in producing L2 strings would be for the teacher to A. correct the students each time they mispronounce a word during oral reading. B. have students read along silently as she plays an audio recording of a short book they are familiar with. She plays the recording once more, with the students reading along chorally. C. make a list of all the words the students mispronounced during reading time and give them a spelling test on this list. D. write simple sentences using words with the L2 phonemes that the students are having trouble with and have the students copy the sentences into their notebooks.

B. have students read along silently as she plays an audio recording of a short book they are familiar with. She plays the recording once more, with the students reading along chorally. Rationale: The teacher needs to promote students' ability to hear the phonemes in L2 contexts. Thus, the audio exercise followed by choral reading of a familiar text would help students boost awareness of target language phonology.

A middle school science class that contains many ESL learners is taught collaboratively by the science teacher and the ESL teacher. The two teachers introduce the topic of earthquakes by leading a class discussion and creating a semantic map. Which of the following skills with ELLs primarily develop when creating their own semantic maps? A. distinguishing between fact and opinion B. organizing and categorizing information C. evaluating the reliability of information D. predicting the results of a sequence of events

B. organizing and categorizing information Rationale: enables students to organize/categorize info based on the key vocabulary terms

Mr. Campbell, a middle-school reading teacher has the following discussion with Aseem about the high school's football game. Mr. Campbell: I saw you at the football game on Friday. Did you have fun? Aseem: Yes, I have fun. I watch brother. Mr. Campbell: Oh, is your brother on the team? Aseem: She...he play the band. He playing drums. Mr. Campbell: Well, that's nice of you to go out and support him. Which of the following does Mr. Campbell practice in the conversation to promote Aseem's speaking development? A. Expanding his use of figurative language to improve expression B. using authentic topics to expand vocabulary C. rephrasing incorrect responses to develop grammar D. providing supportive language cues to prompt responses

B. using authentic topics to expand vocabulary Rationale: incorporates authentic topics to expand student's vocabulary

BICS

BICS Basic Interpersonal Communicative Skills Factors: Casual Practical Social Everyday Context embedded 1 to 2 years to reach native fluency Purpose: Express wants / needs Make jokes Exchange greetings Express agreement or disagreement Make personal conversation

Which of the following statements correctly expresses current understandings about ESL students' acquisition of L2 listening and speaking competencies? A. In order to read and write in L2, students must first acquire L2 oral language. B. If they are motivated to learn, ESL students acquire oral language naturally with little or no need for formal instruction. Proficiency in the other language domains follows. C. In learning L2, proficiency in the reading, writing, speaking, and listening domains develops simultaneously rather than sequentially. D. To help students succeed academically, teachers should focus on reading and writing proficiency.

C. In learning L2, proficiency in the reading, writing, speaking, and listening domains develops simultaneously rather than sequentially. Rationale: ESL theorists and practitioners promote the position that L2 competencies in the speaking, listening, reading, and writing domains develop holistically and simultaneously. Theorists refer to this as the whole-to-part approach which holds that learners construct meaning by processing information contextually. However, this does not mean that learners acquire competency in each area equally. The "opposite" approach—part-to-whole—would have learners acquiring listening skills first, then speaking, then reading, and finally writing. ESL specialists promote learning approaches that focus on the whole communicative context.

A high school English teacher is about to start a unit on Jack London's "To Build a Fire" in her sheltered class. As a follow-up, the teacher takes the class to the school library and assigns the following activity: • Work with your group members to find three facts about Alaskan geography, topography, and/or temperatures. • Be ready to do a three-minute summary of your findings tomorrow. • Your presentation must include at least one photograph. You can show a photograph from a library book, or you and your group members can draw an illustration. Which of the following descriptions best explains how this activity contributes to students' oral-language proficiency development? A. It allows students to connect research-based facts to the events of the story. B. It creates opportunities for students to negotiate about which facts are most relevant to the story. C. It promotes communicative competence by combining collaboration, research, writing, and oral presentation. D. It prepares students to understand the core conflict of the story.

C. It promotes communicative competence by combining collaboration, research, writing, and oral presentation. Rationale: This short research assignment integrates several language domains within the context of content-area knowledge

A third-grade teacher is conducting a phonics lesson in her class of beginning ELL students. She writes the following words on the board, pronouncing each word and having her students respond chorally. She has volunteers come to the board to draw a line under the part that is the same in all the words. She asks for other volunteers to draw a vertical line between the underlined part and the beginning sound(s). Which language-learning strategy does this activity reflect? A. Developing vocabulary B. Applying morphological knowledge in creating new words C. Recognizing patterns in language D. Reinforcing orthographic knowledge

C. Recognizing patterns in language Rationale: This is a classic onset-rime activity designed to reinforce the learners' recognition of patterns in L2 words (a descriptor in the cross-curricular knowledge English Language Proficiency Standards). Onset-rime reinforces students' phonological as well orthographic knowledge.

A high school English teacher plans a unit focused on a frequently taught short story from American literature. Because half of his students are intermediate to advanced ESL students, he needs to provide appropriate accommodations to create comprehensible input. Which of the following instructional activities should the teacher select to meet this goal? A. Before starting the unit, the teacher writes 20 vocabulary words on the board and gives students a class period to look them up. B. Students watch a film on the author's novels and stories, focusing on the shared thematic elements. C. The teacher begins the unit with a "book talk" in which he introduces the characters, the initiating event, and touches on the conflict. He then reads a few pivotal passages from the story. D. For homework prior to the first unit day, students are required to read the story and answer a set of questions.

C. The teacher begins the unit with a "book talk" in which he introduces the characters, the initiating event, and touches on the conflict. He then reads a few pivotal passages from the story. Rationale: This teacher faces the challenge of ensuring that both his native speakers and the ESL students learn literature content and that the ESL learners have appropriate instructional scaffolding to promote language learning. The book talk would provide scaffolding for all learners, including the native speakers.

A fifth-grade teacher wants to promote her intermediate ELL students' understanding of content-area lessons. She has a word wall divided into content areas and each week updates new content vocabulary. She also has content cubicles decorated with posters and realia. Which of the following grouping strategies might further promote her students' content-area learning? A. The teacher uses a random grouping approach, creating new groups each Monday morning. B. The teacher creates two types of groups: one set includes only ELL students, and the other, only native speakers. C. The teacher creates base groups for cooperative learning activities. Each group includes native speakers and ELL students. D. The teacher allows students to self-select the groups they want to be in.

C. The teacher creates base groups for cooperative learning activities. Each group includes native speakers and ELL students. Rationale: This scenario focuses on cooperative learning as a boost to ELL student achievement. Integrating native speakers and ELL in base groups (groups that are maintained for a substantive period rather than just for a specific activity) allows students to bond socially. Those social connections, particularly in an ELL setting, promote learning.

Which of the following descriptions represents the initial procedure for designating students as Limited English Proficient (LEP) in Texas school districts? A. The LEP designation is made by individual teachers any time they notice that students demonstrate deficiencies in social and academic language proficiency. B. When parents request in writing that their child be placed in bilingual or ESL classes, the student is automatically designated LEP. C. Upon entering a school district, any student whose home language is not exclusively English (as determined by a home language survey) is tested to assess language proficiency. Results are evaluated by the Language Proficiency Assessment Committee (LPAC) to determine LEP or non-LEP status. D. Initial LEP designations are based on students' performance in the previous academic year. In the case of kindergarten students, the designation is based on teachers' observations of the learner's performance in the first six weeks of school.

C. Upon entering a school district, any student whose home language is not exclusively English (as determined by a home language survey) is tested to assess language proficiency. Results are evaluated by the Language Proficiency Assessment Committee (LPAC) to determine LEP or non-LEP status. Rationale: This response correctly summarizes the procedure explained in the LPAC Framework Manual. The manual also states that the terms LEP and ELL are used interchangeably in state documents.

A middle school teacher is finishing up a unit on a young adult novel in her ELL class. She plans a culminating activity focused on developing oral-language proficiency in the context of discussing the complexities of the novel. Which of the following assignments would best address her goal? A. Working in groups, students create a poster with illustrations of key events in the novel. B. Each student picks his or her favorite passage from the book and reads it aloud, explaining why the passage is important to the story. C. Working in groups, students create and present a five-minute play focusing on key passages from the book, showing how the central conflict develops. D. Working independently, each student writes and the

C. Working in groups, students create and present a five-minute play focusing on key passages from the book, showing how the central conflict develops. Rationale: A five-minute play allows for complex integration of the four language domains within the context of content-area knowledge. Additionally, this collaborative assignment calls for critical thinking skills (analysis and argument).

According to the Texas Administrative Code (TAC) Chapter 89, the basic requirement for implementing an ESL or bilingual program in a school district is A. a district-wide enrollment of 50 percent or more students whose families are classified as Hispanic, Asian, African American, or other in the U.S. Census Data. B. campus demographics that include 80 percent speakers of languages other than English. C. an enrollment of 20 or more ELL students in any language classification in the same grade level district-wide. D. an audit from the Texas Education Agency that finds disparities in educational approaches used for native and nonnative speakers of English.

C. an enrollment of 20 or more ELL students in any language classification in the same grade level district-wide. Rationale: The 20 or more stipulation comes from the Texas Education Code and is reiterated in the LPAC Framework Manual.

An elementary ESL teacher is planning a reading lesson for ELLs at the beginning and intermediate levels of English language proficiency. Which of the following strategies will most effectively introduce the new academic reading vocabulary? A. providing the students with a list of the new vocab words for them to decode and memorize B. asking the students to write each new vocab word on an index card and then copy the dictionary definition on the back of each card C. discussing with the students the definition of each vocab word and then having them restate the definition in their own words D. having the students write an original sentence for each new vocab word in a journal

C. discussing with the students the definition of each vocab word and then having them restate the definition in their own words

Volunteers from local churches and businesses are interested in assisting students in a school's ESL program. Which TWO of the following tasks are the most appropriate roles for the volunteers to fulfill? A. team teaching with a general education or ESL teacher B. facilitating guided reading group and literature circles C. mentoring students with the guidance of the classroom teacher D. assess with the tutoring of struggling readers and writers E. administering TELPAS and other assessments to gather data

C. mentoring students with the guidance of the classroom teacher D. assess with the tutoring of struggling readers and writers

A high school ESL teacher wants the class to understand the differences between informal language used with friends and formal language used in the classroom. Which of the following strategies will best help ELLs understand the differences? A. asking the ELLs to keep a journal of conversations with friends to be discussed in class B. having the ELLs listen to audio recording in different registers in English C. pairing the ELLs and having them role play a variety of scenarios in the different registers D. reminding the ELLs to use academic language in classroom discussions and informal language in conversations

C. pairing the ELLs and having them role play a variety of scenarios in the different registers

CALP

CALP Factors: Academic Higher level vocabulary Language of education, textbooks, literacy, content areas, formal writing 5 to 7 years to reach native fluency Purpose: Comprehend written text Produce written text Ask/answer informational and clarifying questions related to academic content Make connections involving academic information Conduct research

Vocabulary Acquisition

Can be done through: Instruction • Meanings of new words • Differences between words of similar meanings • Connotations of words • Correct usage of words based on context Read Alouds and Reading Independently • Many new words and concepts in a variety of different genres

Successful Strategies for Teaching Vocabulary

Choose words that: relate to human motivation will generate discussion are critical for literary analysis are difficult for L2 learners have the power to sharply increase comprehension Explain words through: Dramatization, visualization, personification, contextualizing, simplified definition Limit new words. Revisit words over a period of days. Connect words. Categorize words.

1964 Civil Rights Act: Title VI

Civil Rights Act: Title VI -prohibits discrimination on the basis of race, color, or national origin in the operation of all Federally assisted programs

Cognates

Cognates are words from different languages that are spelled almost the same and share similar meanings. • For example, there are many words in English and Spanish that look alike and have similar meanings. • There are three levels of cognates: • True Cognates • Partial Cognates • False Cognates

CALLA Model (Chamot & O'Malley)

Cognitive Academic Language Learning Approach Through carefully designed lesson plans tied to content curriculum, teachers explicitly teach learning strategies and have students apply them to instructional tasks. Plans are based on these assumptions: 1. Mentally active learners are better learners. 2. Strategies can be taught. 3. Learning strategies transfer to new tasks. 4. Academic language learning is more effective with learning strategies.

Content-based Instruction

Communicative approach led to content-based ESL instruction to prepare student for mainstream classes. Taught by ESL educators Addresses key topics in grade-level curriculum Seeks to develop students' English proficiency through study of subject area content Addresses academic skills

Stages of Language Development Holophrastic (8-14 months)

Complete ideas in one word sentences Sound/meaning connected Concerns actions, emotions, naming Can comprehend more than can produce

What is culture?

Culture is the shared beliefs, values, and rule governed patterns of behavior that define a group.

The Texas English Language Proficiency Assessment System (TELPAS) is used to provide which of the following types of data about ELL students? A. Information about the effectiveness of classroom ELL instruction in addressing state standards in writing and reading B. Average yearly progress ratings for ELL students as mandated by the Language Proficiency Assessment Committee C. Correlations between state-mandated exam scores and ELL students' end-of-year grades in math, science, social studies, and English D. Individual proficiency level ratings of ELL students (beginning, intermediate, advanced, advanced high) in listening, speaking, writing, and reading

D. Individual proficiency level ratings of ELL students (beginning, intermediate, advanced, advanced high) in listening, speaking, writing, and reading Rationale: TELPAS includes Proficiency Level Descriptors identifying ELL students according to their current levels of competence and performance in writing, reading, speaking, and listening. The ratings enable teachers to offer differentiated, formative instruction aimed at moving all learners toward the high advanced level.

ELL students in a third-grade class are having trouble learning the names of math figures (for example, hexagon, quadrilateral, pentagon, and so on). Which of the following instructional strategies would most effectively promote students' learning in this area of math? A. The teacher devises a quiz in which students have to correctly match the figure to its name. The quiz is administered every day until all the students get 100 percent correct. B. The teacher draws each figure on the board and has students copy the figures into their notebooks. The teacher asks for volunteers to come to the board to label each figure. C. The teacher creates a poster for each figure. In addition to an illustration of the figure, the name is written in large letters, with the root underlined and the corresponding number of sides written in large print on the poster. The poster includes pictures of words with the same root. D. The teacher puts students into groups and assigns a different figure to each group. Their task is to make several models of their figure using a variety of resources such as craft sticks, twigs, pencils, construction paper strips, chenille sticks, and any other materials they can think of.

D. The teacher puts students into groups and assigns a different figure to each group. Their task is to make several models of their figure using a variety of resources such as craft sticks, twigs, pencils, construction paper strips, chenille sticks, and any other materials they can think of. Rationale: This is the most learner-centered response and most appropriate for the targeted age and ELL level. Working with manipulatives in a math class is considered a learning-inducing strategy: hands-on strategies create engagement and promote understanding.

A non English speaking student from Portugal has just enrolled in middle school in a small rural district with few ELLs. There is one ESL teacher who travels between the elementary, middle school, and high school. Which of the following suggestions will best help the school accommodate the new student? A. giving English Portuguese dictionaries to the teachers who will have students in their class B. requiring a parent to accompany the student to class until the student adjusts to the school routine C. hiring a bilingual aide to translate class content D. arranging volunteer buddies in each class to help the student throughout the school day

D. arranging volunteer buddies in each class to help the student throughout the school day

According to the LPAC Framework Manual, a student is identified as an English Language Learner if he or she A. demonstrates significant deficiencies in writing a short passage in English. B. is unable to respond readily to instructions in English. C. has had no academic experiences in L1. D. is in the process of acquiring English and has a language other than English as a native language.

D. is in the process of acquiring English and has a language other than English as a native language. Rationale: The LPAC Framework Manual stipulates that the defining characteristic of the ELL student is whether a language other than English is spoken at home either exclusively or in addition to English.

Which of the following is a similarity between L1 and L2 acquisition? A. Learners generally acquire and apply rules with equal facility for both B. learners are no more inhibited/anxious in learning an L2 than their L1 C. learners generally have more world knowledge learning L1 than an L2 D. learners construct language from existing concept knowledge in both

D. learners construct language from existing concept knowledge in both

A teacher replies to an incorrect response from an ELL by recognizing the student's effort through positive reinforcement. By recognizing the student's effort, the teacher is demonstrating an understanding of which of the following? A. enhancing linguistic development B. incorporating various learning modalities C. promoting cognitive development D. lowering the affective filter

D. lowering the affective filter Rationale: makes ELL feel appreciated for putting forth the effort

Parental Rights

Districts: Must request parental permission to place student in BE or ESL program Must provide information describing the benefits of the program Parents: Must approve child's placement within the program May deny placement Students: In grades 9-12 may sign their own home language survey (HLS)

Assessment

ESL students may understand significant amounts of the content, but not be able to demonstrate their understanding on a test. Give test orally. Students can often say more than they can read or write. Simplify the language of the test. Try to avoid words you have not taught. Test only the specific skills and concepts you have taught; don't test language. In other words, grade content, not form. Look for evidence of understanding of content. Have students fill in a graphic organizer or label a drawing

Additional Research Findings

ESL taught through academic content is more effective than ESL Pullout. Strongest predictor of L2 student achievement is the amount of formal L1 schooling. The more L1 grade-level schooling, the higher L2 achievement. Bilingually schooled students outperform comparable monolingually schooled students in academic achievement in all subjects, after 4-7 years of dual language schooling.

TEKS format

Example: (8) [Reading=STRAND][Comprehension of Literary Text=COMPONENT][Fiction=subsection] [Students understand, make inferences and draw conclusions about the structure and elements of fiction and provide evidence from text to support their understanding. Students are expected to=KNOWLEDGE AND SKILLS STATEMENT] [(a) sequence and summarize the plots main events and explain their influence on future events; (b) describe the interaction of characters including their relationships and the changes they undergo; and (c) identify whether the narrator or speaker of a story is first or third person=STUDENT EXPECTATIONS]

Fluency

Fluent reading is reading in which words are recognized automatically. Formula for fluent reading: Read and reread decodable words in connected text.

ELLs served by Special Education LPAC Membership vs. ARD*

How is an LPAC different than an ARD? The LPAC can discuss multiple children in the same meeting. The LPAC has a parent representative for all LEP students, not just the individual child's parent. The Bilingual/ESL Program does not need parent permission for testing language proficiency. The key members of the ARD in conjunction with the key members of the LPAC committee determines appropriate assessment for entry, exit and state assessment. The ARD committee in conjunction with the LPAC make all determinations regarding identification and placement for LEP students with disabilities.

How to teach strategies and how to select them

How to teach strategies Develop the students' awareness by providing a variety of activities Strategies are taught explicitly by: Modeling how to use the strategy Naming the strategy Telling students how the strategy will assist them in learning the material Describing when, how, and for what kinds of tasks the strategy can be used Provide ample instructional supports while students practice and apply the strategy Help students be aware of which strategies work for them and why through self evaluation Encourage students to use the strategies in other learning situations How to select strategies The curriculum determines the strategy Start with a small number of strategies Use tasks of moderate difficulty Choose strategies with strong empirical support (ELPS, Marzano) Use strategies that apply to different content areas

Other Language Implications in L2

Idiomatic Language: Ex: "It's raining cats and dogs." Meaning: very heavy rainfall

Acquiring a Second Language for School: Linguistic Development

Includes: Subconscious aspects of language development (innate ability) AS WELL AS... Conscious, formal teaching of language and acquisition of the written system of language

Language Register Examples

Informal Register: The water disappeared. It sucks it up. The car follows the magnet. He is psycho. You're in my bubble. Formal Register: The water evaporated. A vacuum was created because... The magnet force pulled the metal car through the thickness of the desk. He had a nervous breakdown. My personal space is being violated.

Affective Variables

Inside the Classroom Teacher expectations Grouping practices Opportunities Outside the classroom Peer interaction Grouping practices Community characteristics (subtractive or additive bilingualism)

Accommodations for Language Arts

Instructional strategies may include the following objectives: to explain special vocabulary terms in words known to the students to write shorter and less complex sentences to assign short homework tasks that require reading to teach the words that signal sequence to check understanding of written language that may convey complex concepts to rewrite story problems in simpler English by using shorter sentences and pictures to teach new vocabulary in each day's unit and to review terms already mastered to tape short stories for independent listening assignments to de-emphasize speed and emphasize accuracy of reading to help students organize their materials, set realistic goals, and develop independent study habits

Accommodations for Mathematics

Instructional strategies may include the following objectives: to help students build card files and glossaries on mathematics vocabulary to show the same information through a variety of difference graphs and visuals to encourage students to underline key words or important facts in their written assignments to encourage students to underline key words into meaningful groups to pair students for team learning to teach English expressions for mathematical operations such as "square", "add", and "multiply" to teach words that indicate quantitative relationships such as "more", "less", "larger", and twice as many" Texas Education Agency Bilingual/ESL Unit 136 Sept

Accommodations for Science

Instructional strategies may include the following objectives: to practice cause and effect relationships in the environment, laboratory, and on field trips facilitated by providing language and visual cues to teach the special vocabulary of the scientist, particularly verbs such as discover, classify, and hypothesize to help students build notebooks of their hypotheses, materials, procedures, data, and conclusions or experiments and field experiences to ask numerous questions which require higher level thinking responses to limit the number of variables in laboratory experiments to show the same information through a variety of different charts and visuals to develop meanings through the science materials and activities rather than in terms of the equivalent words of the students' vernacular since direct translations often do not convey the exact meaning to stress definitions of terms based on the students' observations to read a variety of sources to highlight contributions of scientists, inventors, and researchers to contrast interrogative, negative, and affirmative statements drawn from the science lesson to encourage careful, thoughtful reading of short selections in which one main idea is presented to encourage students to underline key words or important facts in written assignments to teach interrogative words and expressions and show how they are used in science to answer such questions as who, how, when, and where and higher level questions to encourage complete sentences, correct spelling, and accuracy of expression of science methods and language Texas Education Agency Bilingual/ESL Unit 136 Sep

Accommodations for Social Studies

Instructional strategies may include the following objectives: to help students build individual card files on needed vocabulary for social studies to show the same information through a variety of different graphs and visuals to build vocabulary needed to read maps and legends as these are discussed to encourage students to underline key words or important facts in their written assignments to teach necessary vocabulary for sorting categories of social studies concepts into groups and to explain this vocabulary in words known to the student to use student pairs for team learning (cooperative learning) especially for reports to teach the vocabulary helpful in evaluating material for logic of written expression and for categorizing as opinion or fact to write shorter and less complex sentences and paragraphs with fewer sentences for easier comprehension to use language experience techniques in discussing concepts and ideas to teach the words that signal sequence to check understanding of written language that may convey complex concepts

Brown vs. Board of Education (1954)

Intentional segregation is unequal

Parent Involvement: Parents as Resources

Invite parents to serve as resource speakers to the classroom to share aspects of their culture that apply to a particular curriculum unit such as geography, language, music, system of government, transportation, etc.

Language Objectives Aligned to the ELPS

Language Objectives: aspects of language the teacher is explicitly trying to develop Basic vocabulary of content or academic skills (compare, predict, recommend, etc.) Functional language (discuss in a group, conduct a survey, reach a consensus, request information, etc.) Identify and preteach essential vocabulary

No child left behind 2001

Language instruction for limited English proficient students, Funded under Title III

General Principles of Human Languages

Language is personal (varies by person, topic, purpose, situation, region, social group). Languages are diverse, yet they share many universal properties. Each language uses a finite set of discrete sounds to form words to convey meaning/words combine to form an infinite variety of sentences. Each language is governed by complex rules, many of which are unknown to speakers (linguistic competence). Speech is primary/writing is secondary. Languages change over time/languages are flexible and responsive.

Stages of Language Development Babbling (6 months)

Large variety of sound Learning to distinguish native language sounds Begin to resemble adult speech pitch

1974 Lau v. Nichols (Supreme Court)

Lau v. Nichols (Supreme Court) -San Francisco offered no dual language programs for language learners -Anyone could enroll, everyone had the same classes -Suit by Chinese parents in San Francisco leads to Supreme Court ruling that identical education does not constitute equal education under the Civil Rights Act -School districts must take 'affirmative steps' to overcome educational barriers faced by non-English speakers -Office of Civil Rights interpreted the court's decision as effectively requiring transitional bilingual education unless a school district could prove that another approach would be equally or more effective. -Congress passes the Equal Educational Opportunity Act, extending the Lau decision to all schools

Socioculture/Communicative Theory (Vygotzky)

Learning occurs within the interpersonal space of teacher-student interactions. Temporary support (scaffolding) is essential. Zone of Proximal Development Tasks children can complete independently Tasks children can complete when assisted by a teacher Tasks children cannot complete even with assistance

Categories of Syntax

Lexical Noun (N) Harry, boy, wheat, policy, moisture, bravery Verb (V) arrive, discuss, melt, hear, remain, dislike Adjective (A) good, tall, old, intelligent, beautiful, fond Preposition (P) to, in, on, near, at, by Adverb (Adv) silently, slowly, quietly, quickly, now Nonlexical Determiner (Det) the, a, this, these Degree word (Deg) too, so, very, more, quite Qualifier (Qual) always, perhaps, often, never, almost Auxiliary (Aux) will, can, may, must, should, could Conjunction (Con) and, or, but

Guidelines for Teacher-made Assessments

Look at your test from an ESL student's point of view: Items are clearly written. Directions are easily understood. Examples of how to respond are provided. Readability level of items should be commensurate with the language of the students. Example: A beginning level English speaking student would not be able to read a fifth grade text, so the teacher must take the fifth grade text and align it with appropriate strategies for the beginning student. Test has logical groupings of items of related content. Higher-order thinking questions should be included on every test. Test covers important concepts, not trivia. Teacher grades short answer and essay questions based on student's meaning, not on grammatical errors. Test is not too long for time allowed.

Learning Strategies to Increase CALP

Metacognitive • Planning for learning • Monitoring one's own comprehension and production • Evaluating how well one has achieved a learning objective Cognitive • Manipulating material to be learned mentally (ex: imagery elaborating) • or physically (ex: group items to be learned, taking notes) Social / Affective • Interacting with another person to assist learning (ex: cooperative learning) • Asking for clarification • Using affective control to assist learning tasks

Entry into BE/ESL Program

NEW TO TEXAS SCHOOLS ONLY PK - 1 Qualifies for program if the student: Meets the qualifying criteria on the OLPT indicating limited English proficiency indicated on the TEA List of Approved Test, and Parents have approved and signed the parent permission form. Grades 2 - 12 Qualifies for program if the student: Meets the qualifying criteria on the OLPT indicating limited English proficiency indicated on the TEA List of Approved Tests, or Meets the qualifying criteria on the norm-referenced test indicating limited English proficiency indicated on the TEA List of Approved Tests, and Parents have approved and signed the parent permission form.

Maslow's Hierarchy of Human Needs

Need for safety and security Assign buddies Establish predictable classroom routines Need for a sense of belonging Make personal contact with students - greetings, smiles, eye contact

Identification of ELL Students

Note: Identification and Entry are two distinct steps. PreK - 1st Assessed by TEA approved oral language proficiency test (OLPT) for each student who has a language other than English on the HLS. Grades 2 - 12 Assessed by TEA approved OLPT for each student who has a language other than English on the HLS and Norm-referenced test from TEA list to check literacy and academic achievement.

When should a home language survey be conducted? What two questions should be included?

Only 1 survey per student and only if they are new to the district. It is registered in English or Spanish or home language. 1. which language is spoken in home most of time 2. what long does child use most of the time

Language Instruction Step 2: Use Content-Based Instruction

Opportunities for meaningful communication Concurrent social and cognitive development Wide range of academic concepts and language functions

What is the difference between deep and surface culture?

Part of culture is visible, like the tip of an iceberg. Examples: Language Clothing Food Customs Art Below the surface are the more meaningful and powerful aspects of culture: Beliefs - What we see as truth. Norms - Unwritten rules for behavior. Values - What we hold most important.

Phoneme

Phoneme is a speech sound. There are consonant and vowel phonemes. There are about 25 consonant phonemes or sounds: 18 consonant phonemes, such as /d/ and /t/, are represented by a single letter. 7 phonemes such as /ch/ and /sh/, are represented by two letters. The letters c, q, and x do have a unique phoneme assigned to them. The sounds that they represent are more commonly represented by other letters and spellings; the sound /k/ or /s/ for c, the sounds /kw/ for qu, and the sounds /ks/ for x. Generally, there are about 18 vowel phonemes, or sounds. The letters a, e, i, o, and u are classified as vowels. These 5 letters are used to represent many different sounds. Every syllable has a vowel sound. To produce a vowel sound the air flow is unobstructed by any portion of the mouth.

Phonetics

Phonetics is the study of how phonemes are physically produced. Consonant phonemes may be classified according to: Place of articulation Manner of articulation And whether they are voiced or unvoiced They can be further classified as continuous or stop sounds To produce a consonant phoneme the air flow is cut off either partially or completely. In order to produce a vowel phoneme, the air flow is unobstructed, or continuous. Vowel phonemes are classified as continuous sounds. Tongue Position Is the tongue high, mid, or low in the mouth? Is the tongue near the front, central, or back of the mouth? Mouth Position How rounded are the lips? How tense are the mouth and jaw muscles?

Concepts of Language Systems

Phonology Morphology Syntax Lexicon Semantics Discourse Pragmatics

Lin is an intermediate English speaker, but her teacher has noticed that Lin often interrupts her conversation partners before they are finished with their turn

Pragmatics

Tono is able to speak English fairly fluently, but he often uses slang and other inappropriate words in the classroom. His teacher has decided to create a lesson to help Tono realize and correct his inadvertent impoliteness

Pragmatics/register

ESL Program Components

Programs should address affective, linguistic, and cognitive needs through TEKS-based curriculum including: Oral language development Literacy Grade-level content Academic strategies Assess progress with: TELPAS (language development) STAAR (academic development)

The Writing Process: Editing

Purpose: • Correcting spelling • Grammar • Punctuation • Mechanics, etc. Strategies: • Peer editing groups • Proof reading • Computer programs for spelling • Mini lessons.

The Writing Process: Prewriting

Purpose: • Generating and gathering ideas for writing • Preparing for writing • Identifying purpose and audience for writing • Identifying main ideas and supporting details Strategies: • Discussion/oral activities • Brainstorming, clustering • Questioning, reading, keeping journals in all content areas.

The Writing Process: Drafting

Purpose: • Getting ideas down on paper quickly • Getting a first draft that can be evaluated according to purpose and audience Strategies: • Fast writing • Daily writing • Journals of all types: buddy journals, dialogue journals, learning logs

The Writing Process: Revising

Purpose: • Reordering arguments or reviewing scenes in a narrative • Reordering supporting information • Reviewing or changing sentences Strategies: • Show and not tell • Shortening sentences • Combining sentences • Peer response groups • Teacher conferences.

The Writing Process: Publishing

Purpose: • Share writing with one another, with students, with parents • Showing that writing is valued • Creating a classroom library • Motivating writing Strategies: • Writing may be shared in many formats: • papers placed on bulletin boards • paper published with computers • paper shared in school book fairs, etc.

Higher Order Thinking Skills...

Questioning techniques that require students to analyze, evaluate and synthesis... How does character A compare to character B? Why do you think the character made that choice? What would you have done different if you were the character?

Plessy V. Ferguson 1896

Separate but Equal (segregation) is constitutional

1974 Serna v. Portales

Serna v. Portales -10th circuit court of appeals found that Spanish surnamed students achievement levels were below those of their Anglo counterparts -ordered Portales Municipal Schools to implement a bilingual/bicultural curriculum, revised procedures for assessing achievement and hire bilingual school personnel

Sheltered Instruction

Sheltered Instruction developed to support the needs of English language learners in content area classes. Grade-level curriculum Taught by content-area educators trained in sheltered instruction strategies Uses modified instruction strategies Uses modified instruction to make grade-level content comprehensible

Phonemic Awareness

Spoken words made up of sounds L2 learners should not be involved in phonics instruction that isolates sounds and letters from meaningful use of text. Phonemic awareness develops as children are read to and have opportunities to read; it can also be acquired as children learn to write.

Stages of Development in Communication Skills

Stage 1: One-way communication Learners listen to the new language but do not speak . This is the "silent period". During this period learners are acquiring knowledge about the new language, including vocabulary, syntax, and content knowledge. Stage 2: Partial two-way communication The learners listen to communication and respond with either gestures or their native language. Students can show comprehension without speaking (ex: nodding, pointing, drawing, gesturing). Stage 3: Full two-way communication Learners listen and respond effectively in the target language. Progress through the stages is enhanced when the level of activity matches the learners' stage of development.

Interaction

Students clarify their understanding of new information by talking about it. ESL students need the opportunity to interact with peers to discuss and use new information. Students benefit from using English to express their ideas, opinions, and answers in cooperative groups. Use a variety of grouping strategies.

To Develop Competency in Listening and Speaking

Students need: teachers who understand stages of language acquisition teachers who are tolerant of errors many opportunities to interact with others time.

ELLs served by Special Education LPAC Responsibility for SpEd ELLs

Students with disabilities whose score indicates limited English proficiency on the assessment as determined by the key members of the ARD committee in conjunction with the key members of the LPAC are identified as LEP.

Several of the ELLs in Mr. Walter's class can be heard asking the following questions during an interactive, communicative lesson, not consecutively: Go you to the park everyday? Like you pizza?

Syntax/Phonology

Comprehensible Input

Teachers must make choices of how to communicate information so that it is comprehensible. Oral Language - slower rate of speech, repeat significant information, clearly identify main ideas, use models, give demonstrations Written Information - adapted texts, graphic organizers Strategies - cognates, vocabulary, reading

Content Objectives Aligned to the TEKS

Teachers should be very familiar with TEKS for their course. able to focus English language learners on the most fundamental concepts in a unit or lesson first. knowledgeable about ways to make the content comprehensible.

ELLs served by Special Education LPAC / ARD Collaboration

The ARD committee and the LPAC must work collaboratively. Timeline challenge: Initial Placement Bilingual/ESL timeline - 20 school days TAC §89.1225(g) Special Education - 90 calendar days CFR §300.301(c) and 300.323 Exit Decision End of the year TAC §89.1225(h) The work Determine appropriate participation Assessments for entry and exit Design appropriate instruction Formative assessment Benchmarks/Progress Monitoring Determine how instruction will be provided Measure effect of instruction based on annual goals

Brain Lateralization

The allotment of certain functions to the left or right hemispheres of the brain. Affects language acquisition Left-Logic, literal, grammar & vocabulary, fact retrieval Right-Holistic, pragmatic, contextual, artistic, spatial

Semantic Ambiguity

The individual meaning of words has been resolved, but the context is needed for understanding. Examples: I haven't slept for 10 days. For a duration of 10 days? In the last 10 days? There was not a single man at the party. Not one? Not any that were unmarried?

ELLs served by Special Education LPAC / ARD Coordination

The key members of the LPAC in conjunction with the key members of the ARD can determine that: a student was identified as LEP and does not have any disability; therefore, the student should be served by the Bilingual/ESL education program and NOT referred to special education unless there is data that indicates a disability. a student has a disability identified by the special education program and is also limited English proficient (LEP). The student could be served by both programs, special education and Bilingual/ESL education.

Thomas and Collier Research Findings

The programs that assist students to fully reach their English only peers in both L1 and L2 in all subjects and to maintain that level of high achievement through the end of schooling - and have fewest dropouts were Bilingual Dual Language Programs.

Phonics

The study and use of sound/spelling correspondences to help students identify written words. Phonics instruction teaches students the relationships between letters (graphemes) and speech sounds (phonemes). Phonics instruction can help students learn how to figure out pronunciation of new words that they encounter in print.

Pragmatics

The study of how the meaning conveyed by a word or sentence depends on the context in which it is used. Ex: Such as time, place, social relationship between speaker and hearer, and speaker's assumption about the hearer's beliefs. Examples: The language of a waiter who wants a good tip Politically correct language The language used by a teenager who wants to borrow the car Sarcasm

Morphology

The study of meaningful parts of words and how they are put together. Morphemes are the smallest meaningful spoken units of language. "Morphos" means "form or structure" in Greek; "eme" means "an element or little piece of something."

Communicative Approach (Krashen)

Two separate processes in development of language - acquisition and learning

Meaningful Activities

Ways for ELLL students to process and use new information: take surveys- collate and analyze results produce a product incorporating the new content: news article, play, brochure, poster, poem, illustration conduct an interview and present results make models Teach the learning strategies that are needed for your content. Discuss/model what it means to study for your content area. What does study look like? Teach how to get information from your textbook. Model reading and thinking about the information in the text. Teach how to take notes and use graphic organizers.

Partial Cognates

Words are spelled from the same origin, but the spelling will have a different suffix or prefix. Example: (English) fragrance (Spanish) fragrancia The meaning is the same but the spelling is different and pronunciation will be different.

True Cognates

Words are spelled the same, the meaning is the same. The difference will be in pronunciation and placement of accent. Example: (English) cafeteria (Spanish) cafetería

False Cognate

Words spelled from the same origin, but the spelling will have a different suffix or prefix and the meaning will be different than anticipated according to common origin. Example: (English) exit = to go out or leave (Spanish) éxito = success

creative stage

grade 1 child is able to create his or her own language

communication development

grade 2-6 child acquires difficult phonemes child uses complex grammatical structures

What three critical items should be modified for an ESL program?

instruction pacing materials

Phonology

is the study of the system and pattern of speech sounds of a language

Tonal Register

level of tone of speech (high, medium, low) includes stress. In tonal language, pitch at which syllables in a word are pronounced can make a difference in word meaning. (ex. Thai, Mandarin and dialects of Chinese, Vietnamese, some African languages, some Native-American languages)

Ms. Fuentes is teaching her ELLs to recognize common spelling patterns in English. Today she is focusing on ough and ew.

morpheme

Lexicon

one's mental list of the words in a language, including information about the meaning, grammatical function, pronunciation, etc. (A written lexicon is a list of all the words in a language; dictionary.)

Mr. Smith notices that his asian students often mispronounce words like; learn, leave, law, so he is planning a pronunciation lesson for /l/. He plans to demonstrate the correct pronunciation of words students have already learned that start with that sound. Then he will ask students to mimic his pronunciation while looking at their own mouths in a mirror.

phonemes/phonology

morpheme

smallest unit of language that has meaning

pragmatics

the linguistic term used to refer to the ability to use language in a variety of formal and informal settings to accomplish one's goals

discourse

the linguistic term used to refer to the structure of a language beyond the bounds of a sentence

semantics

the linguistic terms used to refer to the meaning of words

lexicon

the list of words of a language or domain

phoneme

the smallest unit of speech that serves to distinguish one utterance from another

Semantics

the study of linguistic meaning. It studies the meanings of words and ways in which the meanings change and develop. It is concerned with the meanings of words, morphemes, phrases, and sentences. Semantics includes synonyms, antonyms, homonyms, and multiple-meaning words.

phonology

the study of the functions of sounds

morphology

the study of words parts

Cummin's Four Quadrants

theoretical framework that postulates a relationship between language proficiency both in the students native and second language as well as academic achievement

Narrative and Expository Text

• Builds and extends world knowledge about a variety of topics • Extends vocabulary • Connects to real life experiences • Shows how different texts are organized and written • Helps children distinguish different genres and fact from fantasy

Process Writing

• Especially effective for L2 learners because it allows them to write about their own experiences and thus opportunities for L2 development are enhanced. • L2 writers benefit from L1 models and cooperative assistance. • Promote fluency first and then address editing.

Communicative Approach: Monitor

• Grammatical structures acquired in a natural order • Conscious knowledge of rules comes later • With conscious knowledge comes ability to edit (monitor) oral and written language • Must have sufficient time

L2 Literacy

• In early stages of L2 development, developing literacy in L1 is a shortcut to English literacy because we learn to read by reading; it's easier to understand text in L1, and literacy ability transfers. • Once some proficiency in English is achieved, free voluntary reading in English increases English literacy and development of academic English. • There is no reason to stop reading in L1. Continued reading in L1 has cognitive benefits. Also, bilingualism is stronger, and there are life/career benefits.

Guidelines for Scaffolding Discussions

• Model different ways to respond to questions. • Use questions and prompts to help students express their ideas. • Paraphrase and expand students' responses. • Request clarification. • Build world knowledge of the topic and relate to real life experiences. • Promote questions and conversations among students. • Provide positive reinforcement for all types of responses. • Incorporate small group discussions as often as possible to actively involve students.

Language Development Theories

• Nativism / Innatist • Behaviorism • Constructivism / Interactionist • Communicative Approach

Guidelines for Teaching Vocabulary

• Relate words and concepts to personal experiences. • Explain new vocabulary in less complex language. • Highlight vocabulary words by using oral or written cloze procedures. • Categorize words to show relationships. • Help students use visual imagery to remember words.

Strategies and Activities to Develop Fluency

• Repeated Reading - fosters fluent word recognition through multiple exposures to words, and enhances comprehension • Teacher-assisted Reading • Audio-assisted Reading • Computer-based Reading • Partner Reading • Readers' Theater

Communicative Approach: Natural Order

• States that students acquire the rules of language in predictable sequence • According to Lightbrown and Spada (1996, p. 29), developmental sequences are similar across learners from different backgrounds: "What is learned early in one language is learned early by others."

Communicative Approach: Motivation

• The primary reason most of us learn our first language is grounded in the need to belong to the community in which we were born. • A large portion of motivation to learn a second language comes from the same sense of need for acceptance and the desire to interface with the new community or culture.

Communicative Approach: Affective Filter

• This hypothesis states that students must have a risk-free environment in which to acquire and learn a second language. • The needs and emotional states of students will affect whether or not input will be readily available and comprehensible to them.

1978- Rios v Reed

-The Pastchogue-Medford School district's transitional bilingual program was just a course in english. -The federal district courts deemed that the students were denied an equal education opportunity since the students were not receiving academic instruction in Spanish.

Babbling Stage

8 months Cooing and babbling

An ESL teacher pre-teaches the following expressions before having a class discussion: "If I were you," "Why don't you" and "You should." The expressions are most appropriate for practicing which of the following language functions? A. clarifying B. disagreeing C. Interrupting D. Advising

D. advising Rationale: expressions would be most useful when giving advice

When is a district required to offer an ESL program?

if a school district is not required to provide bilingual programs, they can provide an ESL program. district is required to offern an esl program regardless of the students grade level and home language and regardless of the number of such students

Cummins Four Quadrants Examples of quadrant: -Cognitively Undemanding (BICS) -Context Embedded (viewing)

-Following Directions -face to face conversation -getting absence excuse -buying popcorn -oral presentations -content classes (art, music)

TELPAS

-assesses all identified ELLs who are receiving language support skills -spring assessments identify ELLs for the next school year

speech emergence

1-3 years -has good comprehension -can produce simple sentences -makes grammar and pronunciation errors -frequently misunderstands jokes

expansion and delimiting stage

48 months speech has features of adult language

BICS vs. CALP refers to: A. social language vs. academic language B. formal language vs. informal language C. cognitively difficult vs. simple language D. specific subject vs. general vocab

A. social language vs. academic language

Periodically, a high school ESL teacher asks each student to complete the following checklist as a self-assessment tool. Yes or No: I look for word patterns in a sentence to help me read and understand it. I use note taking and flashcards to reinforce new language and vocabulary I have learned. I make word associations when learning new language and vocabulary. I use visualization to help me remember new vocabulary. The primary purpose of the checklist is to help students: A. become effective at determining their own language proficiency. B. develop strategies for overcoming misunderstandings when communicating. C. compensate for gaps in their current language knowledge and skills. D. use various cognitive strategies for internalizing language.

D. Use various cognitive strategies for internalizing language Rationale: Each part of checklist is a strategy students can use to process and internalize language.

For a class that is made up of students from various language backgrounds, the ESL teacher prepares to introduce several vocabulary words. To most effectively introduce the word "actor" the teacher should A. use synonyms of the word "actor." B. translate the word "actor" into the students' native languages. C. model the word "actor" with facial expressions. D. elicit meaning from a sentence with the word "actor."

D. elicit meaning from a sentence with the word 'actor' Rationale: best way for students to comprehend vocab is to actively involve them in creating meaning

When is a district required to offer a bilingual program?

Each school district that has enrollment of 20 or more ELLs in any language classification in the same grade level have to offer a bilingual education program. They will provide a program by offering dual language instruction in pre k through elementary

Ms. Thompson has taught her ELLs that the basic word order of English is a Subject-Verb-Object. She has given pairs of students some words that they can arrange to form sentences. She notices that some students have formed sentences that follow the correct word order but do not make sense. For example, Leo and Raul formed the following sentence: My dog barks food.

Syntax

automatic stage

kinder level child can create original utterances

Syntax

the linguistic term used to refer to the structure of a sentence The study of the way in which sentences are constructed and how sentences are related to each other. Combine phonemes into morphemes, morphemes into words, words into sentences

1920's-1960's

-English immersion or 'sink or swim' policies are dominant method of instruction of language minority students. -Few or no remedial services are available and students are generally held at the same grade level until enough English is mastered to advance in subject areas.

1978- Castaneda v Pickard

-Reputed to be the most significant court decision affecting language minority students after Lau. - In responding to the plaintiffs' claims that Raymondville, Texas Independent School District's language remediation programs violated the Equal Educational Opportunities Act (EEOA) of 1974, the Fifth Circuit Court of Appeals formulated a set of basic standards to determine school district compliance with EEOA. - The "Castaneda Test" includes the following criteria: (1) Theory: The school must pursue a program based on an educational theory recognized as sound or, at least, as a legitimate experimental strategy; (2) Practice: The school must actually implement the program with instructional practices, resources, and personnel necessary to transfer theory to reality; (3) Results: The school must not persist in a program that fails to produce results. A Federal Court decision that impacted language minority students.

1978- Cintron v Brentwood

-The Federal District Court for the Eastern District of New York rejected the Brentwood School District's proposed bilingual program on the grounds that it would violate the 'Lau Guidelines' by unnecessarily segregating Spanish-speaking students from their English-speaking peers in music and art -the court also objected to the program's failure to provide for exiting students whose English language proficiency was sufficient for them to understand mainstream English instruction

1987- Gomez v Illinois

-The Seventh Circuit Court of Appeals ruled that State Education Agencies are also required under EEOA to ensure that language minority students' educational needs are met, by demonstrating that the educational program employed will teach the child English sooner than a program comprised of more extensive Spanish instruction. -A Federal Court decision that impacted language minority students.

Cummins Four Quadrants Examples of quadrant: -Cognitively demanding (CALP) -Talking/Debating

-demonstrations,experiments -basic math computations -plane geometry -projects and activities -health instruction -social studies -science experiments

Cummins Four Quadrants Examples of quadrant: -Cognitively demanding (CALP) -Transforming

-standardized tests -reading/writing -math concepts and applications -concepts -lecture w/ few illustrations -social science texts -mainstream english texts -most content classes

Cummins Four Quadrants Examples of quadrant: -Cognitively Undemanding (BICS) -Context Reduced (doing)

-telephone conversation -note on the refrigerator -written directions, instructions

preproduction

0-1 year -minimal comprehension -does not verbalize -nods yes and no -draws and points

Describe the three needs a bilingual and/or Esl program should address

1. Spanish and English as languages of instruction 2. school districts use stated adopted english and spanish instruction materials and supplementary materials as curriculum tools to enhance the learning process 3. school districts may use other curriculum adoptions that have been developed

advanced

5-7 years -students has near native level of speech

early production

6 months-1year -has limited comprehension -produces one or two words phrases -participates using key words and familiar phrases -uses present tense verbs

Echolalic Babbling Stage

8-12 months cooing and babbling echoes sounds hear in environment

1982 Keyes v. School District #1

A US district court found that a Denver public school district had failed to adequately implement a plan for language minority students (the second element of the Castaneda test)

Educators in the ESL program at an elementary school involve students' families in programs decision-making and support families' participation in other school activities and projects. These practices best reflect an awareness of which of the following factors affecting language development? A. ELLs whose families have positive opinions about school and learning are more likely to develop English-language proficiency B. family members are students are first teachers, and the more they know about language instruction, the better they can teach ELLs specific aspects of language C. Family involvement in school activities provides students with a model of the kind of purposeful communication that is the ultimate goal of language instruction D. ELLs' family members are better able to evaluate the effectiveness of language instruction when they are familiar with the curriculum

A. ELLs whose families have positive opinions about school and learning are more likely to develop English-language proficiency Rationale: parents of ELLs who adopt a positive opinion about school will help encourage their children to have positive attitudes towards learning

Use the information below to answer the questions that follow. The following conversation takes place between Samira, a middle school ESL student, and Mrs. Lynn, her new ESL teacher. Samira is visiting her new school and meeting Ms. Lynn before the first day of school. Ms. Lynn: Good afternoon, Samira. I'm glad you will be in my class this year. Tell me a little about yourself. Samira: Hi! My name is Samira, and I'm from Iraq. I come here with baba, mother, and brother. I being here since two weeks...I don't know how to say...what to say. Nothing. Ms. Lynn: Do you have any hobbies? Samira: I reading poems {pronounces the /p/ as a /b/} Ms. Lynn: Oh, that's nice. I like poems, too. Tomorrow morning can you introduce yourself to the other students? Samira: No...No...Isoshy...very shy. Ms. Lynn: No problem. Well, I'll see you tomorrow! During the first week of school, Ms. Lynn takes anecdotal notes on Samira. Based on her notes she realizes that Samira is inhibited about interacting with other students, which directly affects her oral-language development. When planning lessons or activities for Samira, Ms. Lynn takes Samira's reluctance to speak with others into consideration and sets up an instructional goal to increase her oral-language proficiency through self-initiated interactions with classmates. To best meet the instructional goal, Ms. Lynn should ensure that lessons include which of the following? A. Forming small groups for speaking activities B. Explaining the rules of polite conversation C. Discussing why each class topic is important D. Incorporating independent reading time

A. Forming small groups for speaking activities Rationale: Feels less shy in a small group, encourages her to engage in a discussion

Mr. Gregory, a history teacher, notices within the first two weeks of school that Lana is struggling with history content. When reviewing Lana's prior school records, Mr. Gregory learns that Lana exited the ESL program the previous year at her former school. He decides to meet with the LPAC to discuss Lana's status and performance in school. During the LPAC meeting, Mr. Gregory learns more about Lana's schooling and is able to discuss her performance in his class with the committee. After discussing Lana's performance, the committee sets an objective to follow established LPAC guidelines to help Lana. Which of the following best meets the LPAC'S objective? A. Lana's progress will be monitored for the next two years B. Mr. Gregory will meet with Lana's parents C. Lana will be placed in ESL classes for the next two years D. Mr. Gregory will administer an oral language assessment in Lana's native language

A. Lana's progress will be monitored for the next two years Rationale: Students that have been exited from a language program have to be monitored for 2 years

Mr. Kinu, an elementary teacher, has several intermediate level ELLs in his class. During workstation time, all students participate in language workstations and Mr. Kinu takes anecdotal notes as students work at the stations. Each stations has a stack of note cards with a word written on each card. The following directions are at station one. 1. Work with a partner 2. Make a sentence using these words. 3. You must use all the words. 4. You may not add any words. 5. You and your partner must agree on the sentence. 6. Write the sentence in your notebook. Which of the following is the primary focus of station one? A. Syntax B. Phonology C. Morphology D. Pragmatics

A. Syntax Rationale: helps learn grammatical structures of sentences, phrases, and clauses

According to the concept of the 'Dual Iceberg Representation' regarding L1 and L2 acquisition, which of these is most accurate? A. it is more effective to teach skills and concepts in L1 while teach L2 vocab B. it is more effective to teach skills and concepts in L2 while teaching L2 vocab C. the visible surface features that different from L1 to L2 are the largest components D. the underlying skills for L1 are different from those required for learning the L2

A. it is more effective to teach skills and concepts in L1 while teach L2 vocab

ESL students are learning new English vocabulary words. To which area does this most relate? A. lexicon B. discourse C. pragmatics D. academics

A. lexicon Rationale: refers to vocabulary either in general or in a specific subject or discipline

Rachid, a high school ELL, wrote the following paragraph in his history class. The Kaunstion of the america was writing in 1787. the Kaunstion had creating a three parts for the government. The three parts are a kongress president and a supreme kourt. Also made the system federal of country. Based on Rachid's writing, which of the following accommodations would best help improve his writing? A. providing Rachid with sample sentence stems that are more grammatically complex than his sentences B. allowing Rachid to draw his responses for future writing assignments C. providing Rachid with a list of high frequency words that had been used throughout the year D. assigning Rachid new writing tasks that are above his current level of content understanding evident in his sentences

A. providing Rachid with sample sentence stems that are more grammatically complex than his sentences Rationale: allows him to improve his writing in the context are by providing him with a model

Which of the following strategies would be most effective in helping intermediate and advanced ELLs improve their reading fluency? A. providing frequent opportunities for students to read and reread texts written at their independent reading levels B. expanding students vocabulary knowledge by assigning challenging texts at and beyond their instructional reading levels C. encouraging students to use various comprehension strategies, such as self monitoring, predicting, and questions D. administering timed reading tests to students monthly to motivate them to read more quickly and accurately

A. providing frequent opportunities for students to read and reread texts written at their independent reading levels Rationale: increases reading rate/fluency, increases self efficacy, encourages students to continue to improve

If an ESL student has had no previous education or support in his/her native language, about how long will it usually take to acquire CALP in English after coming to America? A. seven to ten years B. six months to two years C. five years to seven years D. less time than six months

A. seven to ten years

Which of the following best explains why ELLs need to receive direct instruction in the use of nonverbal elements of English? A. the meanings of gestures and body language vary from culture to culture B. people need explicit instruction in nonverbal communication because they lack instinctive communication skills C. cultures associated with English tend to have more taboos related to the body than other cultures D. nonverbal gestures only have meaning when they are connected to specific phrases in the oral language

A. the meanings of gestures and body language vary from culture to culture Rationale: need to understand what various nonverbal elements of English are

A middle school ESL teacher is working with a group of ESL students who English-language abilities vary. Which of the following would be the most appropriate strategy for evaluating the progress of students who are at different proficiency levels in English? A. using multiple measures, such as observations, test scores, and samples of daily work B. selecting language achievement tests that have been normed on a similar student population C. establishing a grading curve and distributing students' test results along the curve D. assessing students only in those areas of English in which they have achieved competence

A. using multiple measures Rationale: allows the teacher to comprehensively assess each students abilities in all 4 domains of language acquistion

Use the information below to answer the questions that follow. The following conversation takes place between Samira, a middle school ESL student, and Mrs. Lynn, her new ESL teacher. Samira is visiting her new school and meeting Ms. Lynn before the first day of school. Ms. Lynn: Good afternoon, Samira. I'm glad you will be in my class this year. Tell me a little about yourself. Samira: Hi! My name is Samira, and I'm from Iraq. I come here with baba, mother, and brother. I being here since two weeks...I don't know how to say...what to say. Nothing. Ms. Lynn: Do you have any hobbies? Samira: I reading poems {pronounces the /p/ as a /b/} Ms. Lynn: Oh, that's nice. I like poems, too. Tomorrow morning can you introduce yourself to the other students? Samira: No...No...Isoshy...very shy. Ms. Lynn: No problem. Well, I'll see you tomorrow! During the first week of school, Ms. Lynn takes anecdotal notes on Samira. Based on her notes she realizes that Samira is inhibited about interacting with other students, which directly affects her oral-language development. When planning lessons or activities for Samira, Ms. Lynn takes Samira's reluctance to speak with others into consideration and sets up an instructional goal to increase her oral-language proficiency through self-initiated interactions with classmates. Which of the follow should Ms. Lynn incorporate to best help Samira to her current level of oral-language proficency? A. Concept attainment model B. Cognitive Academic Language Learning Approach (CALLA) C. Reciprocal teaching strategy D. Total Physical Response (TPR)

B. Cognitive Academic Language Learning Approach (CALLA) Rationale: fosters participation in content directed instruction

Which of the following sentences best demonstrates the use of "must" in an imperative sentence? A. The lights are off; they must not be at home. B. You must not eat candy before sleeping. C. The trip at the zoo yesterday must have been fun. D. You must be tired after such a long journey.

B. You must not eat candy before sleeping. Rationale: imperative command prohibiting someone from doing something

A Texas high school has a program that integrates ESL instruction with academic instruction. The focus of the program is for students to learn English as a second language and use it as a medium to learn other academic subjects. In the program, a full-time teacher provides supplementary instruction for all academic subjects. Based on the characteristics, the program can best be described as... A. sheltered instruction B. content-based ESL C. two-way immersion D. pull-out ESL

B. content-based ESL Rationale: integrates ESL instruction with content area instruction simultaneously

An ESL teacher is instruction new students in which English alphabet letters represent which English speech sounds. This most closely relates to: A. semantics B. phonology C. morphology D. syntax

B. phonology

Students in an ESL class play a game in which they are split into teams. One member of each team picks an index card that has a verb written on it, and that student must act out the verb using movements and gestures. The other members of the team must state what action the student is in the process of doing, such as "Jiawen is driving." The game will most likely help students learn how to correctly form and use the A. simple present tense. B. present progressive tense. C. simple past tense. D. present perfect tense.

B. present progressive tense Rationale: teacher wants them to form sentences with present progressive tense

A middle school ESL teacher regularly includes news and magazine articles in the ESL curriculum that focus on multinational organizations or businesses and highlight careers in which it is advantageous or essential to have knowledge of more than one language. The use of such materials in the ESL program is most beneficial for ELLs because they can help students... A. identify the features of different types of bilingual communities and networks B. recognize the benefits of being bilingual in a global society C. understand the circumstances that may have brought their families to the US D. decide where they would like to live and work when they grow up

B. recognize the benefits of being bilingual in a global society Rationale: understand the global scope of billingualism

According to many ESL experts, which is most accurate about concepts and skills that ESL students have learned in their native languages? A. students must unlearn what they learned in L1 to learn in L2 B. students can transfer these learned skills from their L1 to L2 C. students must learn skills all over again in English as their L2 D. students must learn concepts separately in each language

B. students can transfer these learned skills from their L1 to L2

Mr. Kinu, an elementary teacher, has several intermediate level ELLs in his class. During workstation time, all students participate in language workstations and Mr. Kinu takes anecdotal notes as students work at the stations. Each stations has a stack of note cards with a word written on each card. The following directions are at station one. 1. Work with a partner 2. Make a sentence using these words. 3. You must use all the words. 4. You may not add any words. 5. You and your partner must agree on the sentence. 6. Write the sentence in your notebook. As Mr. Kinu takes anecdotal notes, he realizes that the directions for station one should have an additional step to help students successfully complete the station and to self-assess their work. Based on Mr. Kinu's observation, which of the following is the most appropriate step he should add to the directions? A. Read the sentence out loud. Does it need a comma? B. Write the sentence again without the words "my" and "mom". C. Read the sentence out loud. Does it make sense? D. Write the sentence again and replace 'mom' and 'I' with other words.

C. Read the sentence out loud. Does it make sense? Rationale: questioning if sentences make sense allows students to reflect on work

Ms. Miranda asks the students in her high school English class to write a paragraph describing school in their home countries. The following is a student's sample. I like going to school, I attended school in Cuba. In Cuba, I went to school during the day and I workd at night on homework. I went to school to learn English. I studyed hard before I came to Texas. After reviewing the student's paragraph, Ms. Miranda plans appropriate interventions and follow-up activities for the student. Based on the writing sample, the student's English language proficiency level for writing is best classified as: A. beginning b. intermediate c. advanced d. advanced high

C. advanced Rationale: writes with a basic grasp of english usage and some understanding of complex usage

Ms. Crawford, an ESL teacher, administers a multiple-choice test to her middle school ELLs. The majority of the students do not pass the test. She discusses the test with the students after ward, and they state that they did not understand the directions or the format of the test. Based on the discussion with the students, Ms. Crawford's most appropriate plan of action is to: A. reduce the number of questions on future multiple-choice assessments. B. use performance-based assessments for the remainder of the semester. C. break instructions into steps to be read aloud for future multiple-choice assessments. D. provide future assessments in the students' native languages.

C. break instructions into steps to be read aloud for future multiple-choice assessments Rationale: better understand the test this way

Mr. Kinu, an elementary teacher, has several intermediate level ELLs in his class. During workstation time, all students participate in language workstations and Mr. Kinu takes anecdotal notes as students work at the stations. Each stations has a stack of note cards with a word written on each card. The following directions are at station one. 1. Work with a partner 2. Make a sentence using these words. 3. You must use all the words. 4. You may not add any words. 5. You and your partner must agree on the sentence. 6. Write the sentence in your notebook. To most appropriately accommodate beginning level ELLs in station one, Mr. Kinu should... A. incorporate additional word cards with sight words B. model how to manipulate the words cards C. include corresponding illustrations on the words cards D. allow nonparticipation

C. include corresponding illustrations on the word cards Rationale: can use visuals as a reference point

Ms. Miranda asks the students in her high school English class to write a paragraph describing school in their home countries. The following is a student's sample. I like going to school, I attended school in Cuba. In Cuba, I went to school during the day and I workd at night on homework. I went to school to learn English. I studyed hard before I came to Texas. After reviewing the student's paragraph, Ms. Miranda plans appropriate interventions and follow-up activities for the student. Ms. Miranda can best help the student by focusing on which of the following? A. future tense verbs b. present tense verbs c. past tense verbs d. subject verb agreement

C. past tense verbs Rationale: frequency makes errors when using the past tense

An ESL teacher teaches in a middle school with a diverse student population. In addition to providing her ESL students with language and content instruction, she also helps the students learn how to articulate their feelings, provides them with practice in taking the perspective of others (e.g., through role plays, debates), and encourages the expression of diverse points of view. These practices are particularly effective in: A. recognizing and responding to the linguistic diversity of the students B. promoting students academic achievement C. reducing student conflicts that result from cultural and other misunderstandings D. resolving students cultural identity crises

C. reducing student conflicts that results from cultural and other misunderstandings Rationale: reduces conflict that result from cultural/other misunderstandings

A school district has six elementary schools, all of which enroll a large number of ESL students who come from more than twenty different countries. Which of the following is the most appropriate program model for the district? A. pull-out ESL classes B. early exit billingual C. sheltered English D. late-exit billingual

C. sheltered English Rationale: flexible content based and works to accommodate all levels of ELLS

Mr. Campbell, a middle-school reading teacher has the following discussion with Aseem about the high school's football game. Mr. Campbell: I saw you at the football game on Friday. Did you have fun? Aseem: Yes, I have fun. I watch brother. Mr. Campbell: Oh, is your brother on the team? Aseem: She...he play the band. He playing drums. Mr. Campbell: Well, that's nice of you to go out and support him. Based on the conversation, Aseem demonstrates speaking at the intermediate level of English-Language proficiency because he... A. struggles to connect ideas in sentences B. speaks using common abstract vocabulary C. struggles to use the simple past tense D. speaks using primarily memorized phrases

C. struggles to use the simple past tense Rationale: intermediate speak using simple past tense

Which of the following statements is primarily true about the relationship between a students first and second language acquisition? A. cognitive development in L1 should be discontinued through the elementary years in order to achieve cognitive and academic success in L2 B. cognitive development in L1 should be limited through the elementary years in order to achieve cognitive and academic success in L2 C. students who receive formal schooling in L1 do better at acquiring L2 students with no school in L1 D. students who receive formal schooling in L1 do worse at acquiring L2 then students with no schooling in L1

C. students who receive formal schooling in L1 do better at acquiring L2 students with no school in L1

A school district is implementing a new ESL program. Which of the following is required in order for the program to be in accordance with the criteria established by Castaneda v. Pickard? A. the program must allocate funds to support high quality professional development for all staff that educates ELLs B. the parents of ELLs must be informed of their rights to deny any and all services provided by the program C. the program must be evaluated and found to be effective in achieving its instructional goals in language for ELLs D. the district must ensure that ELLs are not discriminated against because they do not speak English

C. the program must be evaluated and found to be effective in achieving its instructional goals in language for ELLs

A middle school science class that contains many ESL learners is taught collaboratively by the science teacher and the ESL teacher. The two teachers introduce the topic of earthquakes by leading a class discussion and creating a semantic map. A primary reason the lesson is effective for ELLs is that it builds their... A. cross cultural perspectives B. confidence in speaking C. vocabulary development D. attention to detail

C. vocabulary development Rationale: semantic map includes numerous words associated with key vocab term

Acting in his role as advocate for ELLs, an ESL teacher has asked if he could conduct a presentation at an upcoming staff meeting on ways mainstream teachers can improve communication with ELLs. Which of the following kinds of information would be most helpful and appropriate for the ESL teacher to share with colleagues? A. Techniques for introducing and reinforcing new language constructions and how to coach students on pronunciation B. Details about the cultural heritage and religious beliefs of the various ethnic groups represented in the school and how these might impact student achievement C. Suggestions on ways to correct student errors in ways that avoid damaging a student's self-esteem and reward student initiative D. Strategies for clarifying information, checking for comprehension, and controlling the use of complex phrasing, idioms and cultural references

D. Strategies for clarifying information, checking for comprehension, and controlling the use of complex phrasing, idioms and cultural references Rationale: allows mainstream teachers to better ensure they're providing ELLs with comprehensible input

Use the information below to answer the questions that follow. The following conversation takes place between Samira, a middle school ESL student, and Mrs. Lynn, her new ESL teacher. Samira is visiting her new school and meeting Ms. Lynn before the first day of school. Ms. Lynn: Good afternoon, Samira. I'm glad you will be in my class this year. Tell me a little about yourself. Samira: Hi! My name is Samira, and I'm from Iraq. I come here with baba, mother, and brother. I being here since two weeks...I don't know how to say...what to say. Nothing. Ms. Lynn: Do you have any hobbies? Samira: I reading poems {pronounces the /p/ as a /b/} Ms. Lynn: Oh, that's nice. I like poems, too. Tomorrow morning can you introduce yourself to the other students? Samira: No...No...Isoshy...very shy. Ms. Lynn: No problem. Well, I'll see you tomorrow! During the first week of school, Ms. Lynn takes anecdotal notes on Samira. Based on her notes she realizes that Samira is inhibited about interacting with other students, which directly affects her oral-language development. When planning lessons or activities for Samira, Ms. Lynn takes Samira's reluctance to speak with others into consideration and sets up an instructional goal to increase her oral-language proficiency through self-initiated interactions with classmates. Samira's pronunciation error is primarily due to an incorrect... A. place of articulation B. location of the tongue C. manner of articulation D. voicing of the sound

D. Voicing of the sound Rationale: /b/ is voiced and /p/ is voiceless

Task Card Directions: Read the list of phrases below. You are going to listen to a tape of people having conversations. As it is playing, checkmark any of the phrases you hear. -Excuse me, do you know.... -Do you know where... -Have you seen... -Would you mind... -Is there some place where... -Could you tell me.... The instructional activity would be most effective for a teacher to use when introducing ELLs to.... A. acceptable language for continuing different types of conversation B. polite questions to ask when determining another person's social register C. standard methods used to conclude a conversation D. appropriate forms of social requests for information and assistance

D. appropriate forms of social requests for information and assistance Rationale: helps facilitate ESL student's comprehension of new conversational phrases that involve requesting information

A high school has a large populationof ESL students who recently immigrated to the United States. The high school ESL teachers plan to host an orientation for the students and their families to help them better understand the school's culture and protocols prior to the beginning of the school year. In order to most effectively present useful information,and facilitate understanding during the orientation, the teachers should first: A. ask the students to serve as interpreters for their parents. B. provide a detailed analysis of the school district's ESL program. C. organize discussion groups based on the native language of the families. D. ensure that relevant materials are available in the families' native languages

D. ensure that relevant materials are available in the families' native languages Rationale: teachers can ensure the information that is presented is understood

A high school ESL teacher is working with a class of beginning-level ELLs. The teacher asks one student to stand up, then asks another to pick up a pencil. The teacher involves each of the students at different points during the activity. According to proponents of Total Physical Response (TPR), the activity helps students develop English-language skills primarily because it A. encourages them to use English within authentic contexts. B. allows them to discover a wide range of concepts and rules related to English grammar. C. prompts them to use English to accomplish different goals. D. helps them to develop kinesthetic connections to various English words and phrases.

D. helps them to develop kinesthetic connections to various English words and phrases Rationale: kinesthetic connections that students associate with given words/phrases

An ESL teacher provides training to content-area teachers on ways to simplify one's language when talking to ELLs. The training will primarily reinforce the importance of which of the following for the teachers? A. collaborating with other teachers of ELLs B. facilitating parental involvement in students' education C. incorporating community resources within an ESL classroom D. implementing ESL strategies within the classroom

D. implementing ESL strategies within the classroom Rationale: explicit strategies can be used in classes where ELLs are present

Mr. Gregory, a history teacher, notices within the first two weeks of school that Lana is struggling with history content. When reviewing Lana's prior school records, Mr. Gregory learns that Lana exited the ESL program the previous year at her former school. He decides to meet with the LPAC to discuss Lana's status and performance in school. During the LPAC meeting, Mr. Gregory learns more about Lana's schooling and is able to discuss her performance in his class with the committee. After discussing Lana's performance, the committee sets an objective to follow established LPAC guidelines to help Lana. Mr. Gregory can best support Lana's English language acquisition and content understanding by... A. extending the due dates on Lana's content assignments B. implementing the use of a variety of content specific textbooks in Lana's native language C. administering an assessment to determine Lana's current level of content academic language D. implementing strategies to make content comprehensible for Lana

D. implementing strategies to make content comprehensible for Lana Rationale: enables student to better read, write, comprehend, and speak English

Mr. Campbell, a middle-school reading teacher has the following discussion with Aseem about the high school's football game. Mr. Campbell: I saw you at the football game on Friday. Did you have fun? Aseem: Yes, I have fun. I watch brother. Mr. Campbell: Oh, is your brother on the team? Aseem: She...he play the band. He playing drums. Mr. Campbell: Well, that's nice of you to go out and support him. Aseem would most benefit from additional instruction in which of the following? A. subject-verb agreement B. gender pronouns C. comparative adjectives D. verb forms

D. verb forms Rationale: majority of errors

Limited English Proficient Flowchart

FIRST, All students -> Home Language Survey - IF Language spoken at home and by student=English, then = general education -IF language spoken at home and by student=English and any other language, and not English, then = Test is OLPT for Prek-1st grade. OLPT and NRT for 2nd-12th grade. After appropriate test is given and scored..... THEN, LPAC MEETING -IF NON-LEP = general ed classroom -IF LEP = Parental notification If determined LEP and once parents are notified, its placement time. Must determine where student's placement will be: Bilingual Program or ESL program? AFTER, IF Placement = Bilingual Program -Parental Permission = NO, then parent conference, LEP denial, meets exit/reclassification criteria, considered Non-LEP -Parental Permission = YES, spends time in bilingual program, meets exit/reclassification criteria, considered Non-LEP IF Placement = ESL program -Parental Permission = Yes, then put through ESL program, meets exit/reclassification criteria, considered non LEP -Parental Permission= NO, then parent conference, LEP denial, meets exit/reclassification criteria, considered non LEP LASTLY, *LPAC monitors exited students for 2 years after exit criteria met.

What is the difference between deep and surface culture?

Part of culture is visible, like the tip of an iceberg. Language Clothing Food Customs Art Below the surface are the more meaningful and powerful aspects of culture: Beliefs - What we see as truth. Norms - Unwritten rules for behavior. Values - What we hold most important.

1982 Plyler v. Doe (Supreme Court)

Plyler v. Doe (Supreme Court) -Supreme Court denies the state's right to exclude the children of illegal immigrants from public schools -rights for undocumented immigrants -under the 14th amendment of Constitution

1968 The Bilingual Education Act, Title VII of the Elementary and Secondary Act of 1968

The Bilingual Education Act, Title VII of the Elementary and Secondary Act of 1968 -establishes federal policy for bilingual education for economically disadvantaged language minority students -allocates funds for innovative programs -recognizes the unique educational disadvantages faced by non-English speaking students

The ELLs in Mr. Ramirez's class often fail to add an -s or -es to the verb when using 3rd person singular subjects in their writing

morphology

What is the goal of a bilingual and english as a second language program?

to enable ELLs to become competent in listening, speaking, reading, and writing in the English lang through the development of literacy and use of second language methods in all four core classes


Kaugnay na mga set ng pag-aaral

Mental Health Unit 6 Course Point

View Set

Chapter 3: External Environment (Part Uno)

View Set

Fundamentals of Nursing: Chapter 39: Oxygenation

View Set

Business Law I Final Exam Olivet Nazarene University

View Set

Drugs & Behavior - Class #11 (Hallucinogens)

View Set